GRAY'S ANATOMY Week 7

¡Supera tus tareas y exámenes ahora con Quizwiz!

B(. Hyperkyphosis is characterized by a "hunchback" due to an abnormal increase in curvature of the thoracic region of the vertebral column. Hyperlordosis, or "swayback," is an increase in lumbar curvature of the spine. Lordosis can be physiologic, such as seen in a pregnant woman. Scoliosis is a lateral curvature of the spine with rotation of the vertebrae. Spina bifida is a neural tube defect characterized by failure of closure of the vertebral arch. Osteoarthritis is a degenerative disorder that affects the articular cartilage of joints and is not specifically related to the thoracic region of the spine. GAS 76; N 153; McM 97)

1 A 55-year-old man with severe coughing is admitted to the hospital. Radiologic examination is consistent with tuberculosis of the right lung, with extension to the thoracic vertebral bodies of T6 and T7, producing a "gibbus deformity." Which of the following conditions is most likely also to be confirmed by radiologic examination? A. Hyperlordosis B. Hyperkyphosis C. Scoliosis D. Spina bifida E. Osteoarthritis

E(. The ligamentum avum lies within the ver- tebral canal on the anterior aspect of the vertebral arches connecting the lamina of adjacent vertebrae. Puncturing this ligament allows the needle to enter into the epidural/extradural space for the injection of the anesthetic. Although the posterior longitudinal ligament lies within the spinal canal, it will not be punctured during the procedure. The supraspinous ligament connects and passes along the tips of the vertebral spinous processes. The interspinous liga- ment lies between adjacent spinous processes. The anterior longitudinal ligament connects the anterior aspect of the vertebral body. These ligaments do not lie within the vertebral canal. GAS 80-84; N 159; McM 97)

100 A 38-year-old woman has been in labor for 14 hours and has agreed to have an epidural anesthetic injection for pain control. Which of the following structures is most likely to be the last penetrated by the needle before it reaches the epidural space? A. Supraspinous ligament B. Interspinous ligament C. Anterior longitudinal ligament D. Posterior longitudinal ligament E. Ligamenta flava

B(. During a breech delivery as described here, downward traction is applied to the shoulders and upper limbs as the baby is forcibly extracted from the birth canal. This exerts traction on the upper cord of the brachial plexus, often causing a traction injury from which the baby can often recover. If the ventral rami of C5 and C6 are avulsed from the spinal cord, the injury is permanent. GAS 738, 747; N 416; McM 31)

101 A 22-year old pregnant woman was admitted urgently to the hospital after her baby had begun to appear at the introitus. The baby had presented in the breech position, and it had been necessary to exert considerable traction to complete the delivery. The newborn is shown in Fig. 6-7. Which of the following structures was most likely injured by the trauma of childbirth? A. Radial nerve B. Upper trunk of the brachial plexus C. Lower trunk of the brachial plexus D. Median, ulnar, and radial nerves E. Upper and lower trunks of the brachial plexus

A (. The denticulate ligament is a sheet of pia mater running longitudinally on either side of the spinal cord, connecting it to the dura mater. Medially, the denticulate ligament lies between the origin of the anterior and posterior rootlets serving as a landmark to differentiate between them. The conus medullaris is the terminal end of the spinal cord and the lum terminale is an extension of the pia mater that con- nects the conus medullaris to the dural sac. The posterior longitudinal ligament lies posterior to the vertebral bodies, while the ligamentum avum con- nects the lamina of adjacent vertebrae. GAS 99-104; N 165; McM 96)

102 A 7-year-old boy is undergoing a surgery to remove a tumor from his spinal cord. During surgery of the spinal cord, which of the following structures is used as a landmark to identify anterior rootlets from posterior rootlets? A. Denticulate ligament B. Filum terminale C. Conus medullaris D. Posterior longitudinal ligament E. Ligamenta flava

D(. The alar ligament connects the dens to the medial surface of the occipital condyles. It limits excessive rotation of the atlanto-axial joints. Flexion and extension of the upper cervical spine occur at the atlanto-occipital joints and the zygapophysial joints. Lateral exion (abduction) as a combination move- ment at the uncovertebral joints (of Luschka). These are not limited by the alar ligament (GAS Fig. 2-20B). GAS 71-72; N 23; McM 85)

103 A 45-year-old male driver involved in a motor vehicle crash was taken to the emergency department and MRI revealed a complete tear of the right alar ligament. None of the other ligaments of the upper cervical spine were torn. Upon physical examination, which of the following cervical spine movements will be most likely increased as a result of the tear? A. Flexion B. Extension C. Lateral flexion D. Rotation E. Abduction

A(. Axillary-subclavian vein thrombosis is be- coming much more common in recent years because of the extensive use of catheters in cancer patients and other chronic medical conditions. Effort-induced thrombosis is seen with strenuous use of the domi- nant arm with hyperabduction and external rotation of the arm or backward and downward rotation of the shoulder as in playing cricket, volleyball, or baseball, or chopping wood. Because the symptoms of subcla- vian stenosis are fairly dramatic, most patients pre- sent promptly, usually within 24 hours. They complain of a dull ache in the shoulder and axilla, the pain worsened by activity. Conversely, rest and elevation often relieve the pain. Patients with catheter-associated axillary-subclavian deep vein thrombosis report simi- lar symptoms at the arm or shoulder on the side with the indwelling catheter. GAS 759; N 420; McM 206)

105 A 36-year-old man is admitted to the emergency department with a dull ache in his shoulder and axilla (Fig. 6-8). During physical examination the pain worsens by activity, and, conversely, rest and elevation relieve the pain. History reveals that the patient was hospitalized 2 days ago and a central venous line was used. What is the most likely diagnosis? A. Axillary-subclavian vein thrombosis B. Compression of C5 to C8 spinal nerve C. Disc herniation of C4 to C8 D. Impingement syndrome E. Injury to radial, ulnar, and median nerves

B(. During surgery the long thoracic nerve was damaged which supplies the serratus anterior muscle. During abduction of the arm, serratus anterior ele- vates and laterally rotates the scapulae to allow for full abduction, such as when the ability to lift the hand above the head. The serratus anterior is responsible for protracting the scapula and therefore holding it against the thoracic wall. If the nerve supply to this muscle is damaged this will not be achieved when the patient pushes her hands against the wall, resulting in what is called a "winged scapula". GAS 726; N 413; McM 103)

105 A 38-year-old woman with a history of carcinoma of the left breast and who had had a lumpectomy 2 years previously, presents to her surgeon with com- plaints of a hard mass in the left breast. On ultrasound examination, a 3 cm × 4 cm hard mass is discovered in the upper outer quadrant extending in the axillary tail (of Spence). A radical mastectomy is performed successfully and the tumor is removed. Three weeks postoperatively the patient complains of dif culty raising her left arm above her head. Which of the fol- lowing is most likely expected to be found during phys- ical examination? A. Protraction of the both scapulae B. Protrusion of the medial border of the left scapula when hands are pushed against the wall C. Weak abduction of the left upper limb in the 15 to 90 degree range D. Weak retraction of the scapula E. Weak adduction of the humerus

C(. The injury is at the second part of the axil- lary artery. The suprascapular artery is a branch of the thyrocervical trunk off the subclavian artery, prox- imal to the axillary artery. The subscapular artery is the major branch of the third part of the axillary artery, giving off the thoracodorsal and the circum ex scapular arteries. In this case blood would be owing from the circum ex scapular artery in a retrograde direction into the axillary artery, supplying blood distal to the injury. GAS 733-735; N 420; McM 134)

106 A 22-year-old woman had suffered a severe knife wound to the upper lateral portion of her pectoral region, with entry of the knife at the deltopectoral groove. Pressure applied to the wound had prevented further profuse bleeding. In the emergency department, vascular clamps were applied to the axillary artery, proximal and distal to the site of injury, which had occurred between the second and third parts of the axillary artery. The vascular surgeon knew there was time to repair the wound of the artery because of the rich collateral pathway provided by the anastomoses between which of the following arteries? A. Transverse cervical and suprascapular B. Posterior circum ex humeral and profunda brachii C. Suprascapular and circum ex scapular D. Supreme (superior) thoracic and thoracoacromial E. Lateral thoracic and suprascapular

A(. During development the spinal cord lls the vertebral canal entirely. Due to differential growth of the vertebral column and the spinal cord, the cord ends at L3 in an infant. It gradually changes its position to the level of L1/L2, which is the adult level. S2 is the level at which the dural sac normally terminates. GAS 99-110; N 161; McM 97)

106 A 2-month-old infant is admitted to the emergency department with symptoms of meningitis. A lumbar puncture is performed in order to examine the CSF to confirm the diagnosis. The needle is inserted into the lumbar cistern (dural sac). At which vertebral level will the conus medullaris typically be found in this patient? A. L3 B. L4 C. L5 D. S1 E. S2

C(. The winged scapula results from a lesion of the long thoracic nerve, which supplies the serratus anterior muscle. This muscle is responsible for rotat- ing the scapula upward, which occurs during abduc- tion of the arm above the horizontal. The long thoracic nerve arises from the ventral rami of C5 to C7 of the brachial plexus. The diaphragm is supplied by the phrenic nerve, which comes from the ventral rami of C3 to C5 (mnemonic: C3, 4 and 5 keep the diaphragm alive). GAS 727; N 413; McM 129)

108 A 55-year old male firefighter is admitted to the hospital after blunt trauma to his right axilla. Examina- tion reveals winging of the scapula and partial paralysis of the right side of the diaphragm. Which of the fol- lowing parts of the brachial plexus have been injured? A. Cords B. Divisions C. Ventral rami D. Terminal branches E. Trunks

A(. An epidural anesthetic procedure is per- formed in the epidural space which contains fat and the internal vertebral (Batson's) plexus. A hematoma in this region would cause compression on the spinal nerves and possibly the cord resulting in severe pain and de cits. The great anterior medullary artery of Adamkiewicz is the largest of the spinal segmental arteries and is usually located at around T10, much higher than L2 to L3. The anterior and posterior spinal arteries are located in the anterior median and pos- terolateral ssures of the spinal cord, respectively, and are not located in the epidural space. The external vertebral plexus is located external to the vertebral canal and a hematoma of this plexus will not produce the symptoms of this patient. GAS 100-104; N 166; McM 97)

109 A 22-year-old pregnant woman underwent epidural anesthesia in anticipation of labor. After delivery she developed back pain and right lower extremity weakness. Imaging revealed a hematoma in the epidural space resulting in compression of the nerve that exits at the level of L2 to L3. Which of the following vessels is most likely responsible for the hematoma? A. Internal vertebral plexus B. Great radicular artery (of Adamkiewicz) C. Anterior spinal artery D. Posterior spinal artery E. External vertebral plexus

D(. Lymph from the skin of the anterior chest wall primarily drains to the axillary lymph nodes. GAS 748; N 412; McM 179)

11 A 22-year-old man is diagnosed with metastatic malignant melanoma of the skin over the xiphoid process. Which lymph nodes receive most of the lymph from this area and are therefore most likely to be involved in metastasis of the tumor? A. Deep inguinal B. Vertical group of superficial inguinal C. Horizontal group of superficial inguinal D. Axillary E. Deep and superficial inguinal

A(. Ligaments serve to restrict movement. The anterior longitudinal ligament courses downward on the anterior surface of the vertebral bodies attaching to the intervertebral discs along its way. It is stretches from the base of the skull inferiorly to the anterior surface of the sacrum. The anterior longitudinal liga- ment is the most anteriorly positioned ligament of the vertebral column and limits its extension. The poste- rior longitudinal ligament travels on the posterior surface of the vertebral bodies attaching to the inter- vertebral discs along the way. This ligament serves to prevent excessive exion of the vertebral column and extends from C2 to the sacrum. The interspinous liga- ments attach adjacent spinous processes to each other from C2 to the sacrum, it restricts the degree of sepa- ration of the spinous processes during exion. Liga- mentum ava attaches the internal surfaces of adjacent laminae to each other and prevents them from pulling apart during exion. Intertransverse ligaments connect adjacent transverse processes and prevent excessive rotation. GAS 80; N 159; McM 94)

110 Idling at a stoplight in his vintage car without headrests, a 71-year-old-man's car is struck from behind by a truck. The man is brought to the emergency department suffering from a severe hyperextension neck injury due to the crash. The T2-weighted MRI shows a rupture of the anterior anulus fibrosus of the C4 to C5 intervertebral disc and a prevertebral hematoma which compromised his airway and required intubation. Which of the following ligaments is most likely disrupted in this injury? A. Anterior longitudinal ligament B. Posterior longitudinal ligament C. Ligamentum flavum D. Interspinous ligament E. Intertransverse ligament

D(. A cervical rib (usually found at C7) may cause thoracic outlet syndrome, which is a condition characterized by weak muscle tone in the hand and loss of radial pulse when the upper limb is abducted above the shoulder. The mechanism of injury with the gun being red overhead suggests a lower trunk injury to the brachial plexus. The axillary artery supplies the shoulder muscles, and there is no loss of function to these muscles. The upper trunk of the brachial plexus also supplies innervation to the shoulder muscles, which are unaffected based on the patient's present- ing abnormalities. The subclavian artery is located anterior to the brachial plexus until the plexus sepa- rates into cords as it passes under the clavicle. The brachiocephalic artery and lower trunk of the brachial plexus is only partially correct; the brachiocephalic artery is not directly associated with the brachial plexus due to its location at the midline of the body behind the sternum. GAS 150; N 183, 416; McM 129)

111 A 32-year-old man who is an expert target shooter reports pain in his right upper limb and slight tingling and numbness of all digits of the ipsilateral hand. However, the tingling and numbness of the fourth and fth digits is the most severe. The man states that the problem usually occurs when he is ring his gun with his hand overhead. Radiographic studies reveal the presence of a cervical rib and accessory scalene mus- culature. Which of the following structures is most likely being compressed? A. Axillary artery B. Upper trunk of brachial plexus C. Subclavian artery D. Lower trunk of brachial plexus E. Brachiocephalic artery and lower trunk of brachial plexus

A(. The order of structures pierced during an epidural procedure is skin, subcutaneous tissue, muscle, supraspinous ligament, interspinous liga- ment, and ligamentum avum (there is often a midline gap in the ligamentum avum). The anterior longitu- dinal ligament is anterior to the vertebral body and cannot be reached by this approach. The posterior longitudinal ligament is posterior to the vertebral body and can also not be reached by this procedure. The intertransverse ligaments are too lateral and may not be perforated by this technique. GAS 100-104, 115-116; N 166; McM 97)

111 An anesthesiologist administers epidural anes- thetic immediately lateral to the spinous processes of vertebrae L3 and L4 of a pregnant woman in labor. During this procedure, what would be the last ligament perforated by the needle in order to access the epidural space? A. Ligamentum flavum B. Anterior longitudinal ligament C. Posterior longitudinal ligament D. Interspinous ligament E. Intertransverse ligament

B(. The anterior and posterior spinal arteries do not provide suf cient blood supply to the spinal cord below cervical levels and will receive additional supply segmentally along its course from multiple sources. The largest of these vessels are usually termed the artery of Adamkiewicz and arises at the lower thoracic or upper lumbar region. GAS 100-104; N 167; McM 94)

113 A 62-year-old man visits his physician for his annual medical check-up. During physical examination it is noted that the patient has noticeable pulsations on palpation of the lower abdomen. Ultrasound examination reveals a large abdominal aortic aneurysm. The patient is operated on and during the repair his aorta is temporarily clamped. Which of the following arterial anastomoses will most likely prevent ischemia of the spinal cord if the blood pressure drops dangerously low? A. Segmental arteries from the vertebral, intercostals, superficial epigastric, lumbar, and medial sacral arteries B. Segmental arteries from the vertebral, intercostal, lumbar, spinal anterior, and posterior and lateral sacral arteries C. Anterior and posterior spinal arteries D. Radicular arteries of the vertebral, lumbar, intercostal, lateral sacral arteries, and artery of Adamkiewicz E. Segmental arteries from vertebral and intercostals

A. (The superior trunk of the brachial plexus includes C5 and C6, which give rise to the supra- scapular nerve, which innervates the supraspinatus muscle. The supraspinatus muscle is the primary muscle involved in abduction of the arm from 0 to 15 degrees. The deltoid muscle, supplied primarily by C5, abducts the arm from 15 to 90 degrees. The middle trunk is just C7 and has nothing to do with the muscle involved in initial abduction of the arm. The inferior trunk is C8-T1 and does not supply the supraspinatus muscle; therefore, it is not the right answer. The cords are distal to the branching of the supraspinatus muscle; therefore, neither lateral cord nor medial cord is the correct answer. GAS 738, 747; N 416; McM 28)

118 A 34-year-old woman is admitted to the emer- gency department after a car crash. Radiographic stud- ies show marked edema and hematoma of the arm, but there are no fractures. During physical examination the patient presents with inability to abduct her arm with- out rst establishing lateral momentum of the limb, and inability to ex the elbow and shoulder. Which of the following portions of the brachial plexus is most likely injured? A. Superior trunk B. Middle trunk C. Inferior trunk D. Lateral cord E. Medial cord

C(. The radial nerve acts to extend the forearm at the elbow. This nerve is derived from all the ventral rami of the brachial plexus C5 to T1. None of the other answers includes all the ventral rami and are therefore incorrect. GAS 745-746; N 416; McM 96)

119 A 22-year-old man is admitted to the hospital after a car collision. Radiographic examination reveals an oblique fracture of his humerus. Upon physical exami- nation the patient is unable to extend his forearm. The damaged nerve was most likely composed of bers from which of the following spinal levels? A. C5, C6 B. C5, C6, C7 C. C5, C6, C7, C8, T1 D. C6, C7, C8, T1 E. C7, C8, T1

C(. The long thoracic nerve innervates the ser- ratus anterior, which is responsible for elevation and protraction of the scapula beyond the horizontal level while maintaining its position against the thoracic wall. Along with the thoracodorsal nerve, the long thoracic nerve runs super cially along the thoracic wall and is subject to injury during a mastectomy procedure. The axillary nerve, the spinal accessory nerve, and the thoracodorsal nerve supply the deltoid muscle, trapezius muscle, and latissimus dorsi muscles, respectively. The dorsal scapular nerve is responsible for innervation of the rhomboids and levator scapulae. Aside from the long thoracic and thoracodorsal nerves, the remaining nerves do not course along the lateral thoracic wall. GAS 726; N 180, 413; McM 140)

12 A 39-year-old woman complains of an inability to reach the top of her head to brush her hair with her right hand. History reveals that she had undergone a mastectomy procedure of her right breast 2 months earlier. Physical examination demonstrates winging of her right scapula. Which nerves were most likely damaged during surgery? A. Axillary B. Spinal accessory C. Long thoracic D. Dorsal scapular E. Thoracodorsal

A(. The suprascapular artery arises as a major branch of the thyrocervical trunk from the subclavian artery. It has rich anastomoses with the circum ex scapular artery and could provide essential blood supply to the scapula. The dorsal scapular artery would be lost with the graft. None of the other vessels listed is in position to provide adequate supply to the scapula. GAS 720; N 414; McM 31)

120 A 56-year-old woman is admitted to the hospital after a severe car crash. A large portion of her chest wall needed to be surgically removed and replaced with a musculo-osseous scapular graft involving the medial border of the scapula. Which of the following arteries will most likely recompensate the blood supply to the entire scapula? A. Suprascapular B. Dorsal scapular artery C. Posterior circum ex humeral artery D. Lateral thoracic E. Supreme thoracic artery

C(. The triceps brachii muscle is innervated by the radial nerve (primarily C7), which comes off C5 to T1 spinal nerves. Because the patient's only motor de cit involves the triceps brachii muscles, one can rule out C5 and C6, which supply bers to the axillary, musculocutaneous, and upper subscapular nerves. Damage to either of these ventral rami would result in additional motor de cits of the shoulder and exor compartment of the arm. One can also rule out C8-T1 because these ventral rami form the medial pectoral nerve and the medial brachial and antebrachial cuta- neous nerves. Damage to these ventral rami would result in loss of pectoral muscle function and cutane- ous sensation over the medial surface of the upper limb. GAS 745, 761; N 416; McM 96)

125 A 45-year-old woman is admitted to the hospital with neck pain. An MRI examination reveals a herni- ated disc in the cervical region. Physical examination reveals weak triceps brachii muscle. Which of the fol- lowing spinal nerves is most likely injured? A. C5 B. C6 C. C7 D. C8 E. T1

B(. Injury to the superior trunk of the brachial plexus can damage nerve bers going to the supra- scapular, axillary, and musculocutaneous nerves. Damage to the suprascapular and axillary nerves causes impaired abduction and lateral rotation of the arm. Damage to the musculocutaneous nerve causes impaired exion of the forearm. A winged scapula would be caused by damage to the long thoracic nerve. The long thoracic nerve is formed from spinal cord levels C5, C6, and C7, so the serratus anterior muscle would be weakened from the damage to C5 and C6, but the muscle would not be completely paralyzed. The intrinsic muscles of the hand are innervated by the ulnar nerve, which would most likely remain intact. Paraesthesia in the medial aspect of the arm would be caused by damage to the medial brachial cutaneous nerve (C8-T1; inferior trunk). Loss of sensation on the dorsum of the hand would be caused by damage to either the ulnar or radial nerves (C6 to T1). GAS 738, 747; N 416; McM 31)

127 A 42-year-old woman is admitted to the hospital with injury to the upper (superior) trunk of the brachial plexus. The diagnosis is Erb-Duchenne palsy. Which of the following conditions is expected to be present during physical examination? A. Winged scapula B. Inability to laterally rotate the arm C. Paralysis of intrinsic muscles of the hand D. Paraesthesia in the medial aspect of the arm E. Loss of sensation in the dorsum of the hand

C(. The dermatome that encompasses the nipple is supplied by spinal nerve T4. In this case the herpes zoster virus is harbored in the dorsal root ganglion of T4 and can be activated to cause the characteristic rash that is distributed along the derma- tome including the nipple. GAS 139-141; N 162; McM 178)

129 A 29-year old patient complains of sever pain radiating across her back and chest. Upon clinical examination you observe a rash characteristic of herpes zoster infection passing from her upper left back and across her left nipple. Which of the following spinal nerve roots sheds the active virus? A. Dorsal root of T3 B. Ventral root of T3 C. Dorsal root of T4 D. Ventral root of T4 E. Dorsal root of T5

D(. The rotator cuff muscles are common sites of damage during shoulder injuries. These muscles include the supraspinatus, infraspinatus, teres minor, and subscapularis (SITS). Initiation of abduction of the humerus (the rst 15 degrees) is performed by the supraspinatus, followed by the deltoid from 15 to 90 degrees. Above the horizontal, the scapula is rotated by the trapezius and serratus anterior muscles, causing the glenoid fossa to turn superiorly and allowing the humerus to move above 90 degrees. The teres major and the pectoralis major are responsible for medial rotation and adduction of the humerus. These muscles are therefore not involved in abduction at the gleno- humeral joint. GAS 711-712, 717; N 134-135; McM 411, 413)

13 A 19-year-old man is brought to the emergency department after dislocating his shoulder while playing football. Following treatment of the dislocation, he cannot initiate abduction of his arm. An MRI of the affected shoulder shows a torn muscle. Which muscle was most likely damaged by the injury? A. Coracobrachialis B. Long head of the triceps brachii C. Pectoralis minor D. Supraspinatus E. Teres major

B(. The brachioradialis re ex is elicited by tapping the tendon of the brachioradialis muscle. The re ex involves spinal nerves C5, C6, and C7. The major contribution is from C6. GAS 785-787; N 432; McM 150)

137 A 54-year-old woman is found unconscious in her bed. She is admitted to the hospital, and during physical examination she has absence of her brachioradialis reflex. The ventral ramus of which spinal nerve is responsible for this reflex? A. C5 B. C6 C. C7 D. C8 E. T1

C (. Spina bifida is a developmental condition resulting from incomplete fusion of the vertebral arches within the lumbar region. Spina bifida occulta commonly presents asymptomatically with midline, lumbar, cutaneous stigmata such as a tuft of hair and a small dimple. More severe forms (spina bifida cystica) are categorized into three types: Spina bifida cystica with meningocele presents with protrusion of the meninges through the unfused vertebral arches; spina bifida with myelomeningocele is characterized by protrusion both of the meninges and central nervous system (CNS) tissues and is often associated with neurologic deficits; and rachischisis, also known as spina bifida cystica with myeloschisis, results from a failure of neural folds to fuse and is characterized by protrusion of the spinal cord or spinal nerves and meninges. GAS 74; N 14, 160; McM 77, 88)

14 A 1-year-old girl is brought to the clinic for a routine checkup. The child appears normal except for a dimpling of the skin in the midline of the lumbar region with a tuft of hair growing over the dimple. What is this relatively common condition that results from incomplete embryologic development? A. Meningomyelocele B. Meningocele C. Spina bifida occulta D. Spina bifida cystica E. Rachischisis

A(. Somatic afferents are responsible for convey- ing pain, pressure, touch, temperature, and proprio- ception to the CNS. Afferent bers carry only sensory stimuli, whereas efferent bers convey motor infor- mation. Visceral innervation is associated with the autonomic nervous system. Visceral afferents gener- ally carry information regarding the physiologic changes of the internal viscera whereas visceral effer- ents deliver autonomic motor function to three types of tissue: smooth muscle, cardiac muscle, and glan- dular epithelium. GAS 32-35; N 174; McM 211)

15 A young resident complains of an itch on his back that appears to be caused by an insect bite. Which nerve bers carry the sensation of a mosquito bite on the back, just lateral to the spinous process of the T4 vertebra? A. Somatic afferent B. Somatic efferent C. Visceral afferent D. Visceral efferent E. Somatic efferent and visceral afferent

A(. The lateral quadrants of the breast drain into the anterior axillary (pectoral), which is approxi- mately 75% of the lymphatic drainage. The medial quadrants drain into the parasternal nodes with some drainage to the parasternal nodes of the opposite breast. There is also a small amount of drainage of the inferior part of the breast into lymph nodes of the anterior abdominal wall. The central and apical axil- lary nodes receive lymphatic from the pectoral, lateral, and posterior axillary nodes. GAS 140-141; N 182; McM 179)

150 A 35 year old woman patient has a hard tumor ,about 1 cm in diameter slightly above, and lateral, to her right areola. A speci c dye was injected into the tissue around the tumor which was taken up by the lymph vessels, draining the area. An incision was made to expose the lymphatic vessels which were then visible to the naked eye. The vessels were traced to surgically expose the lymph nodes receiving the lymph from the tumor. Which nodes will most likely rst receive lymph from the tumor? A. Anterior axillary (pectoral) B. Lateral axillary C. Parasternal D. Central axillary E. Apical (infraclavicular)

E(. The ulnar nerve is responsible for cutaneous innervation to the medial one and a half digits and motor innervation to most of the intrinsic muscles of the hand including the interossei. The interossei muscles are responsible for adduction of the digits, which is the action that would be used to hold a piece of paper between the ngers. The median nerve sup- plies cutaneous innervation to the lateral three and a half ngers and the thenar eminence and lateral two lumbricals. These muscles function to oppose the thumb and ex the MP joints, respectively. The mus- culocutaneous nerve is responsible for innervation of the anterior compartment of the arm, and muscular nerve bers of this nerve would not be damaged by a wound in the distal forearm. The radial nerve sup- plies the dorsum of the hand, with sensation and extension function of the forearm muscles, and damage will not lead to this array of symptoms (GAS Fig. 7-109). GAS 814, 815; N 464; McM 159)

150 A 36-year-old man is brought to the emergency department because of a deep knife wound on the medial side of his distal forearm. He is unable to hold a piece of paper between his ngers and has lost sensa- tion from the fth digit and the medial side of the fourth digit. Which of the following nerves is most likely injured? A. Axillary B. Median C. Musculocutaneous D. Radial E. Ulnar

B(. The muscle that attaches into the intertuber- cular sulcus of the humerus is the latissimus dorsi. Nerve supply is via the thoracodorsal nerve, which is a branch of the posterior cord and is made up of roots C6-8. Nerves C2, C3 and C4 are not part of the bra- chial plexus but of the cervical plexus and will supply the "strap" muscles. Nerves C4 and C5 are the main contributions to the phrenic nerve, and C5 does not contribute to the formation of the thoracodorsal nerve. GAS 728; N 416; McM 115)

151 A 28-year-old telephone company worker falls off a street pole during a telephone line repair and lands directly on his right shoulder. Plain radiographs reveal a vertical fracture through the entire length of the oor of the intertubercular sulcus of the right humerus. The muscle that is most likely affected by the fracture is innervated by a nerve that is composed of which of the following nerve roots? A. C3 and C4 B. C6 to C8 C. C4 and C5 D. C2 to C4 E. C5 to C7

C(. Anterior dislocation of the humerus may damage the nerves located in the axilla or cause tears in the rotator cuff muscles. Internal rotation is the primary function of subscapularis muscle; with this being the only action impaired it is the most likely damaged muscle, probably as a result of injury to the upper and/or lower subscapular nerves that innervate this muscle. The infraspinatus and trees minor muscles are external rotators, and the supraspinatus muscle is the abductor of the arm from 0 to 15 degrees. The pectoralis major is a exor, adductor, and medial rotator and would not likely be damaged during a shoulder dislocation. GAS 712; N 411; McM 136)

152 A 21-year-old woman who is an athlete dislocated her glenohumeral joint while playing soccer and the shoulder was reduced in the emergency department. However, after 1 week the physician noted that the woman had lost strength when she attempted internal rotation of her arm at the shoulder. This finding was most likely caused by a tear in which of the following muscles? A. Infraspinatus B. Pectoralis minor C. Subscapularis D. Supraspinatus E. Teres minor

C(. The long thoracic nerve arises from the upper three ventral rami to the brachial plexus (C5 to C7) and supplies the serratus anterior, which protracts the scapula. The diaphragm is innervated by the phrenic nerve, which also arises from ventral rami (C3-C5). GAS 727, 741; N 413; McM 140)

158 A 55-year-old man is admitted to hospital after blunt trauma at the junction of his neck and shoulder on the right side. Examination reveals winging of the scapula and partial paralysis of the right side of the diaphragm. Which part of the brachial plexus has been injured? A. Cords B. Divisions C. Ventral rami D. Terminal branches E. Trunks

E(. CSF is found within the subarachnoid space and is continuous with the ventricles of the brain (CSF flows from the ventricles to the subarachnoid space). The epidural space, positioned between the dura mater and periosteum, contains fat and the internal vertebral venous plexus (of Batson). The subdural space, between the arachnoid mater and dura mater, exists only as a potential space and does not contain cerebrospinal fluid. The anterior and posterior longitudinal ligaments traverse the length of the vertebral bodies. GAS 106, 114-116; N 166; McM 97)

16 A 15-year-old woman was suspected of having meningitis. To obtain a sample of cerebrospinal fluid by spinal tap in the lumbar region (lumbar puncture), the tip of the needle must be placed in which of the following locations? A. In the epidural space B. Between anterior and posterior longitudinal ligaments C. Superficial to the ligamentum flavum D. Between arachnoid mater and dura mater E. In the subarachnoid space

B(. The condition described in this patient is called "winging" of the scapula. "Winging" of the scapula occurs when the medial border of the scapula lifts off the chest wall when the patient pushes against resistance, such as a vertical wall. The serratus ante- rior muscle holds the medial border of the scapula against the chest wall and is innervated by the long thoracic nerve. The serratus anterior assists in abduc- tion of the arm above the horizontal plane by rotating the scapula so that the glenoid fossa is directed more superiorly. GAS 727; N 413; McM 141)

16 A 55-year-old man is examined in a neighborhood clinic after receiving blunt trauma to his right axilla in a fall. He has dif culty elevating the right arm above the level of his shoulder. Physical examination shows that the inferior angle of his right scapula protrudes more than the lower part of the left scapula. The right scapula protrudes far more when the patient pushes against the examiner's hand with resistance. Which of the following neural structures has most likely been injured? A. The posterior cord of the brachial plexus B. The long thoracic nerve C. The upper trunk of the brachial plexus D. The site of origin of the middle and lower subscapular nerves E. Spinal nerve ventral rami C7, C8, and T1

A(. Injuries to superior parts of the brachial plexus (C5-C6) usually result from an excessive in- crease in the angle between the neck and shoulder during a dif cult delivery. Injury to the superior trunk of the plexus is apparent by the characteristic position of the limb ("waiter's position"), in which the limb hangs by the side in medial rotation. Injuries to the lower trunk of the brachial plexus (Klumpke paraly- sis) are much less common. These events injure the inferior trunk of the brachial plexus (C8 and T1) and may avulse the roots of the spinal nerves from the spinal cord. The short muscles of the hand are af- fected, and a claw hand results (GAS Fig. 7-52A). GAS 738; N 452; McM 159)

160 Following a difficult delivery, a 3-day-old infant girl showed limited movement of the right upper limb, with the arm adducted and internally/medially rotated, the forearm extended at the elbow and pronated, and the wrist slightly flexed. Tearing of fibers in which ventral rami of the brachial plexus best accounts for these symptoms? A. C5 and C6 B. C6 and C7 C. C7 and C8 D. C8 and T1 E. C5 to T1

D(. The supraspinatus is innervated by the suprascapular nerve (C5, C6) and the nerve continues through the spinoglenoid notch and innervates the infraspinatus. The supraspinatus initiates abduction of the arm up to the rst 15 to 20 degrees. The sub- scapular nerve supplies the subscapularis and teres major muscles, which are medial rotators of the arm. The axillary nerve supplies the deltoid and teres minor muscles and also a patch of skin on the lateral side of the shoulder. The deltoid abducts the arm beyond 20 degrees, and the teres minor muscle, although a lateral rotator, does not abduct the arm. The radial nerve supplies muscles in the posterior compartments of the arm and forearm, which are extensors of the elbow, wrist, and ngers in that order. The upper subscapular nerve supplies the subscapu- laris, a medial rotator of the arm. GAS 717, 742; N 413; McM 138)

164 A 36-year-old woman is admitted to the emer- gency department after an athletic injury that has caused weakness in both lateral rotation and the initial 15 to 20 degrees of abduction of the arm. Which nerve was most probably injured? A. Lower subscapular B. Axillary C. Radial D. Suprascapular E. Upper subscapular

D(. In thoracic outlet syndrome—sometimes caused by a cervical rib or a cervical band—ventral rami or trunks of the brachial plexus can be com- pressed by these structures as they travel from the neck to the axilla. In this case the inferior trunk of the brachial plexus is being compressed by a cervical rib. The anterior division of the inferior trunk contin- ues as the medial cord of the brachial plexus. The medial brachial cutaneous nerve (medial cutaneous nerve of the arm) and medial antebrachial cutaneous nerve (medial cutaneous nerve of the forearm) are branches of the medial cord of the plexus, with the ulnar nerve as its terminal branch. Additionally, there is medial cord contribution to the median nerve. Com- pression of the inferior cord of the brachial plexus therefore presents with numbness and paraesthesia on the medial part of the arm, forearm, and hand. GAS 150; N 415; McM 140)

168 A 25-year-old woman experiences numbness and tingling in her right arm and hand while carrying a piece of luggage. Physical examination showed no motor or sensory de cits in the upper limb. When asked to abduct her upper limb to 90 degrees and to maintain this position while repeatedly closing and opening her hands, the symptoms are reproduced along the medial border of the limb, from the axilla to the hand. Which nerve structure(s) is/are most likely compressed? A. Ulnar nerve at the medial epicondyle B. Radial nerve at the neck of the radius C. Median nerve in the carpal tunnel D. Inferior trunk of the brachial plexus E. Divisions of the brachial plexus

B(. The posterior longitudinal ligament is the only ligament spanning the posterior aspect of the vertebral bodies and intervertebral discs. With intervertebral disc herniation, the nucleus pulposus of the intervertebral disc protrudes posterolaterally. The anterior longitudinal ligament traverses the anterior side of the vertebral bodies and thus would not protect the spinal cord from direct compression. The supraspinous and ligamentum flavum ligaments connect the spinous processes and the laminae of adjacent vertebrae, respectively. The nuchal ligament is a continuation of the supraspinous ligaments near the C7 vertebrae and runs to the occipital protuberance. GAS 80-81; N 159; McM 94)

17 A 19-year-old man is diagnosed with a herniated disc but he has no symptoms of spinal cord injury. In the event of intervertebral disc herniation in the cervical region, which of the following ligaments is in an anatomic position to protect the spinal cord from direct compression? A. Supraspinous B. Posterior longitudinal C. Anterior longitudinal D. Ligamentum flavum E. Nuchal ligament

E(. The injury being described is also known as Erb-Duchenne paralysis or "waiter's tip hand" and is relatively common in children after a dif cult deliv- ery. This usually results from an injury to the upper trunk of the brachial plexus, presenting with loss of abduction, exion, and lateral rotation of the arm. The superior trunk of the brachial plexus consists of spinal nerve ventral rami C5-6. GAS 738, 747; N 416; McM 139)

18 After a forceps delivery of an infant boy, the baby presents with his left upper limb adducted, internally rotated, and exed at the wrist. The startle re ex is not seen on the ipsilateral side. Which part of the brachial plexus was most likely injured during this dif cult delivery? A. Lateral cord B. Medial cord C. Ventral rami of the lower trunk D. Ventral ramus of the middle trunk E. Ventral rami of the upper trunk

C(. This is the location of the conus medullaris, a tapered conical projection of the spinal cord at its inferior termination. Although the conus medullaris rests at the level of L1 and L2 in adults, it is often situated at L3 in newborns. The cauda equina and lum terminale extend beyond the conus medullaris. GAS 99, 100; N 161; McM 97)

18 In spinal anesthesia, the needle is often inserted between the spinous processes of the L4 and L5 verte- brae to ensure that the spinal cord is not injured. This level is safe because in the adult the spinal cord usually terminates at the disc between which of the following vertebral levels? A. T11 and T12 B. T12 and L1 C. L1 and L2 D. L2 and L3 E. L3 and L4

D(. Compression on the inferior trunk of the brachial plexus compresses nerves C8 and T1. These nerves contribute to the medial cutaneous nerve of the arm (C8, T1) and the medial cutaneous nerve of the forearm (C8, T1). They also contribute to the median, medial pectoral, ulnar, and radial nerves. This patient has thoracic outlet syndrome, which causes compression of the inferior trunk of the bra- chial plexus usually by the presence of a cervical rib. Compression of the ulnar nerve at the medial epicondyle, radial nerve at the neck of the radius, or median nerve in the carpal tunnel would cause motor de cits not present in this patient. GAS 738, 747; N 416; McM 129)

182 A 25-year-old woman experiences numbness and tingling in her right arm and hand while carrying a piece of luggage. Physical examination showed no motor or sensory de cits in the upper limb. When asked to abduct her upper limb to 90 degrees and to maintain this position while repeatedly closing and opening her hands, the symptoms are reproduced along the medial border of the limb, from the axilla to the hand. Which nerve structure(s) is/are most likely compressed? A. Ulnar nerve at the medial epicondyle B. Radial nerve at the neck of the radius C. Median nerve in the carpal tunnel D. Inferior trunk of the brachial plexus E. Divisions of the brachial plexus

B(. The radiograph shows a fracture of the humerus at the surgical neck. The bruising and dimpling of the upper arm would result from this injury. The axillary nerve leaves the brachial plexus as a terminal branch of the posterior cord. It passes through the quadrangular space and wraps around the head of the humerus on its way to provide inner- vation to the teres minor, the deltoid, and the portion of skin over the lower aspect of the deltoid that is known as the "sergeant's patch." The radial nerve travels in the radial groove along the shaft of the humerus and would be injured in a fracture of the shaft of the humerus. The ulnar nerve would be injured in a fracture of the medial epicondyle. The median nerve travels too deep to be injured here and could be compressed at the carpal tunnel or at the cubital fossa. The musculocutaneous nerve is likewise within the tissue and will not be affected by this injury. GAS 704-705; N 418; McM 136)

185 A 67-year-old woman with osteoporosis injured her left shoulder/arm in a fall. Examination reveals bruising and dimpling of the upper part of the arm with exquisite tenderness over the affected area. The shoulder radiograph is shown in Figure 6-13. Which nerve is most likely to be injured? A. Radial B. Axillary C. Ulnar D. Median E. Musculocutaneous

A(. The sympathetic division of the autonomic nervous system is primarily responsible for vasocon- striction. Separation of ventral or dorsal roots would lead to undesired consequences, such as a loss of motor or sensory activity. Similarly, surgical division of spinal nerves would also have unwanted conse- quences, but such are not related to the increased arterial constriction and the painful ischemia in the digits. Division of selected sympathetic chain ganglia, however, would decrease the sympathetic out ow to the upper limbs. GAS 38-39; N 163; McM 94-95)

19 A 22-year-old woman is diagnosed with Ray- naud's disease. In such a case, the patient suffers chronic vasospasm in response to cold. This can lead to arterial constriction and painful ischemia, especially in the ngers or toes. Relief from the symptoms in the hands would require surgical division of which of the following neural elements? A. Lower cervical and upper thoracic sympathetic bers B. Lower cervical and upper thoracic ventral roots C. Lower cervical and upper thoracic dorsal roots D. Lower cervical and upper thoracic spinal nerves E. Bilateral spinal accessory nerves

A(. The lower subscapular nerve arises from the cervical spinal nerves 5 and 6. It innervates the sub- scapularis and teres major muscles. The subscapularis and teres major muscles are both responsible for adducting and medially rotating the arm. A lesion of this nerve would result in weakness in these motions. The axillary nerve also arises from cervical spinal nerves 5 and 6 and innervates the deltoid and teres minor muscles. The deltoid muscle is large and covers the entire surface of the shoulder, and contributes to arm movement in any plane. The teres minor muscle is a lateral rotator and a member of the rotator cuff group of muscles. The radial nerve arises from the posterior cord of the brachial plexus. It is the largest branch, and it innervates the triceps brachii and anco- neus muscles in the arm. The spinal accessory nerve is cranial nerve XI, and innervates the trapezius muscle, which elevates and depresses the scapula. The ulnar nerve arises from the medial cord of the brachial plexus and runs down the medial aspect of the arm. It innervates muscles of the forearm and hand. GAS 714-717; N 413; McM 142)

2 A 27-year-old man was admitted to the emer- gency department after an automobile collision in which he suffered a fracture of the lateral border of the scapula. Six weeks after the accident, physical exami- nation reveals weakness in medial rotation and adduc- tion of the humerus. Which nerve was most likely injured? A. Lower subscapular B. Axillary C. Radial D. Spinal accessory E. Ulnar

B(. A crush fracture is characterized by compression of the entire vertebral body. The wedge fracture is similar in that it affects the vertebral bodies, but it involves small fractures around the perimeter of the vertebral body. Both of these fractures cause reductions in overall height. Fracture of the spinal, transverse, or superior articular processes can be due to an oblique, transverse, or comminuted fracture. Intervertebral discs are associated with disc herniation, not compression fractures. GAS 82; N 153-154; McM 88, 107)

2 A 68-year-old man is admitted to the hospital due to severe back pain. Radiologic examination reveals severe osteoporosis of the vertebral column, with compression fractures to vertebrae L4 and L5. Which of the following parts of the vertebrae are most likely to be fractured in this patient? A. Spinous process B. Vertebral bodies C. Transverse process D. Superior articular process E. Intervertebral disc

C(. The anterior longitudinal ligament runs along the anterior-most aspect of the vertebral column from C1 to the sacrum and would therefore be unaffected by a laminectomy. Denticulate ligaments extend laterally from the pia mater to the arachnoid mater along the length of the spinal cord. The ligamentum flavum is one of the two ligaments found in the vertebral canal and is adherent to the anterior aspect of the vertebral arches and often greatly thickened in spinal pathology. It is thus simultaneously removed upon excision of the lamina. The nuchal ligament is a thick longitudinal extension continuing from the supraspinous ligament at the level of C7 to the external occipital protuberance (inion). The cruciate ligament is an incorrect answer because it is located anterior to the spinal cord, and thus would not be involved in laminectomy. GAS 80; N 159; McM 98)

21 A 42-year-old woman is diagnosed with stenosis of the cervical vertebral canal. A laminectomy of two vertebrae is performed. Which of the following ligaments will most likely also be removed? A. Anterior longitudinal B. Denticulate C. Ligamentum flavum D. Nuchal E. Cruciate

C(. The vertebral canal is the longitudinal canal that extends through the vertebrae, containing the meninges, spinal cord, and associated ligaments. The internal vertebral venous plexus is the mostly valveless network of veins extending longitudinally along the vertebral canal. Neither of these answer choices describes a specific space. The spinal epidural space is found superficially to the dura mater. It is a fat-filled space extending from C1 to the sacrum. The subarachnoid space is a true space containing CSF. It is found within the CNS and extends to the level of S2. The subdural space is a potential space between the dura and the arachnoid mater. Normally, these two layers are fused due to the pressure of CSF in the subarachnoid space. GAS 103-110; N 166; McM 94, 96)

22 A 28-year-old pregnant woman is admitted to the obstetrics department for delivery. In the final stages of labor, a caudal anesthetic is administered via the sacral hiatus. Into which of the following spaces in the sacral canal is the anesthetic placed? A. Vertebral canal B. Vertebral venous plexus C. Epidural space D. Subarachnoid space E. Subdural space

B(. The anterior spinal artery is located anteriorly along the spinal cord and is not directly associated with the vertebrae. The vertebral arteries run through the transverse foramina of cervical vertebrae C6 through C1 and are therefore most closely associated with injury to the transverse processes. The ascending cervical artery is usually a very small branch from the thyrocervical trunk of the subclavian artery, running on the anterior aspect of the vertebrae. The deep cervical artery arises from the costocervical trunk and is also a very small artery and courses along the posterior aspect of the cervical vertebrae. The posterior spinal arteries are adherent to the posterior aspect of the spinal cord. GAS 100; N 167; McM 94-95)

24 A 25-year-old male racing car driver is admitted to the emergency department after a severe car crash. Radiologic studies reveal damage to the tip of the transverse process of the third cervical vertebra, with a significantly large pulsating hematoma. What artery is the most likely to have been damaged? A. Anterior spinal artery B. Vertebral artery C. Ascending cervical artery D. Deep cervical artery E. Posterior spinal arteries

B(. Scoliosis is defined as a lateral deviation of the spinal column to either side. Hyperkyphosis is an increased primary curvature of the spinal column. This curvature is associated with thoracic and sacral regions and is most likely this patient's clinical condition. Spinal stenosis is a narrowing of the vertebral canal and is not directly associated with a displacement of the spinal column. Hyperlordosis is the increased secondary curvature affecting the cervical and lumbar regions. A herniated disc is a rupture of the anulus fibrosus of the intervertebral disc, commonly causing a posterolateral displacement of the nucleus pulposus into the vertebral canal. GAS 75; N 153; McM 87)

25 A 79-year-old man, a retired military veteran, presents to the outpatient clinic with an abnormal curvature of the vertebral column. He complains that it has become increasingly painful to walk around town. Upon physical examination, he has an abnormally increased convexity to his thoracic curvature resulting from osteoporosis. Which of the following is the most likely clinical condition of this patient's spine? A. Scoliosis B. Hyperkyphosis C. Spinal stenosis D. Lordosis E. Herniated disc

B (. The anterior longitudinal ligament runs on the anterior aspect of the vertebrae and is not affected. The transverse ligament of the atlas anchors the dens laterally to prevent posterior displacement of the dens. This ligament has been torn in this injury. The ligamentum flavum is found on the posterior aspect of the vertebral canal and does not contact the anteriorly placed dens. The supraspinous ligament is located along the spinous processes of the vertebrae. The nuchal ligament is a longitudinal extension of the supraspinous ligament above the level of C7. GAS 80; N 159; McM 97)

27 A 53-year-old man was in a head-on vehicle collision that resulted in compression of his spinal cord by the dens (odontoid process) of the axis, with resulting quadriplegia. Which of the following ligaments was most probably torn? A. Anterior longitudinal ligament B. Transverse ligament of the atlas C. Ligamentum flavum D. Supraspinous ligament E. Nuchal ligament

C(. The pedicles are bony structures connecting the vertebral arches to the vertebral body. The ligamentum flavum runs on the posterior aspect of the vertebral canal and is more closely associated with the laminae than to the pedicles of the vertebrae. The nuchal ligament is a longitudinal extension of the supraspinous ligament from C7 to the occiput, both running on the most posterior aspect of the vertebrae along the spinous processes. The cruciform (also called cruciate or transverse ligament of the atlas) ligament is a stabilizing ligament found at the skull base and C1/C2. It attaches to the pedicles and helps stabilize the dens. The posterior longitudinal ligament extends the length of the anterior aspect of the vertebral canal and is anterior to the pedicles. GAS 68-69; N 23; McM 85)

29 A 23-year-old man was killed in a high-speed motor vehicle collision after racing his friend on a local highway. When the medical examiner arrives at the scene, it is determined that the most likely cause of death was a spinal cord injury. Upon confirmation by autopsy, the medical examiner officially reports that the patient's cause of death was a fracture of the pedicles of the axis (C2). Breaking of which of the following ligaments would be most likely implicated in this fatal injury? A. Ligamentum flavum B. Nuchal ligament C. Cruciform ligament D. Posterior longitudinal ligament E. Supraspinous ligament

C(. The ligamentum flavum connects the laminae of two adjacent vertebrae and forms the posterior wall of the vertebral canal. It is the only answer choice that is in direct contact with the vertebral foramen. Therefore, hypertrophy of only the ligamentum flavum would present as spinal canal stenosis. The supraspinous and interspinous ligaments connect spinous processes. The anterior longitudinal ligament connects the anterior portion of the vertebral bodies and intervertebral discs. Finally, the nuchal ligament is a thickened extension of the supraspinous ligament above the level of C7. GAS 84; N 159; McM 99)

3 A 45-year-old man is admitted to the hospital because of severe pain in the back and lower limb. Radiologic examination reveals spinal canal stenosis syndrome. Which of the following conditions is most likely to be confirmed by a magnetic resonance imaging (MRI) examination? A. Hypertrophy of supraspinous ligament B. Hypertrophy of interspinous ligament C. Hypertrophy of ligamentum flavum D. Hypertrophy of anterior longitudinal ligament E. Hypertrophy of nuchal ligament

C (The herpes zoster virus after initial infection can remain latent in the dorsal root ganglion until the body becomes immunocompromised or stressed, and then reappears as a vesicular rash along one of the dermatomes, in this condition called shingles. The dorsal root ganglion sensory neurons are derivative of neural crest cells. Other neural crest cell derivatives include the pia mater, chromaffin cells of the adrenal medulla, thyroid parafollicular cells, Schwann cells, melanocytes, cranial nerves, connective tissue, and some bones of the skull and face. There are usually 21 pairs of denticulate ligaments, which take their origin from the pia mater and attach to the dura mater. As the pia mater is derived from neural crest cells the denticulate ligaments take their origin from the pia mater, which dictates that they share the same embryologic origin. The dorsal horn, ventral horn, and conus medullaris are all part of the spinal cord and are notochord derivatives. The dura mater is of mesenchymal origin. 252-253, 266)

31 A 60-year-old man has been feeling "pins and needles" and some sharp pains over his right upper chest and back for several weeks. A rash of red erupted vesicles is seen at the right border of the sternum, a few centimeters above the nipple. Antiviral treatment is initiated to treat herpes zoster. The patient recovers, is free of pain, and his skin looks normal. Which of the following structures has the same embryologic origin as the location where the viral particles are still residing? A. Dorsal horn B. Ventral horn C. Dorsal root ganglion D. Conus medullaris E. Dura mater

D(. The internal vertebral plexus (of Batson) surrounds the dura mater in the spinal epidural space; hence the bleeding would cause the hematoma in that space. The subarachnoid space, containing the CSF, is located between pia and arachnoid mater. A subarachnoid hemorrhage would most likely result from a ruptured intercerebral aneurysm. A subdural hematoma would result most likely from a venous bleed from a torn cerebral vein as it enters the superior sagittal venous sinus within the skull. The central canal is located within the gray matter of the spinal cord. The lumbar cistern is an enlargement of the subarachnoid space between the conus medullaris of the spinal cord and the caudal end of the subarachnoid space. GAS 102; N 169; McM 108)

31 Following a car crash, a 47-year-old woman complains of severe headache and back pain. Radiologic examination reveals bleeding of the internal vertebral venous plexus (of Batson), resulting in a large hematoma. In what space has the blood most likely accumulated? A. Subarachnoid space B. Subdural space C. Central canal D. Epidural space E. Lumbar cistern

E(. The medial cord has been injured by traction on the lower trunk of the brachial plexus. The medial cord is the continuation of the inferior (lower) trunk of the brachial plexus, which is formed by C8 and T1. C5 and C6 are typically associated with the superior (upper) trunk level and thus the lateral cord. C7 forms the middle trunk. An injury to the posterior cord would usually involve the C7 spinal nerve. This is a typical Klumpke paralysis. GAS 738-747; N 416; McM 139)

33 As she fell from the uneven parallel bars, a 17-year-old female gymnast grasped the lower bar briefly with one hand but then fell painfully to the floor. An MRI examination reveals an injury to the medial cord of the brachial plexus. Which of the following spinal nerve levels would most likely be affected? A. C5, C6 B. C6, C7 C. C7, C8 D. C7, C8, T1 E. C8, T1

A(. The atlantoaxial joint is a synovial joint responsible for rotation of the head, not flexion, abduction, extension, or adduction. The atlanto-occipital joint is primarily involved in flexion and extension of the head on the neck. GAS 71; N 19; McM 85)

34 A 22-year-old male soccer player is forced to leave the game following a head-to-head collision with another player. He is admitted to the hospital, and radiologic examination reveals slight dislocation of the atlantoaxial joint. As a result, he experiences decreased range of motion at that joint. What movement of the head would most likely be severely affected? A. Rotation B. Flexion C. Abduction D. Extension E. Adduction

D(. The internal vertebral plexus (of Batson) lies external to the dura mater in the epidural space. To aspirate excess blood, the physician must pass the needle through the ligamentum flavum to reach the epidural space wherein the blood would accumulate. The spinal cord, pia mater, and arachnoid mater are located deep to the epidural space. GAS 102; N 169; McM 97)

35 A 42-year-old man is struck in the back, rupturing the internal vertebral venous plexus (of Batson). Radiologic studies reveal a hematoma causing compression of the spinal cord. When aspirating the excess blood, the physician performing the procedure should stop the needle just before puncturing which of the following structures? A. Spinal cord B. Pia mater C. Arachnoid mater D. Dura mater E. Ligamentum flavum

C(. The radial nerve is the most likely nerve com- pressed to cause these symptoms. This type of nerve palsy is often called "Saturday night palsy." One reason for this nickname is that people would sup- posedly fall asleep after being intoxicated on a Satur- day night with their arm over the back of a chair or bench, thereby compressing the nerve in the spiral groove. The radial nerve innervates all of the exten- sors of the elbow, wrist, and ngers. It innervates the triceps brachii muscle but the motor branch typically comes off proximal to the site of compression, so the patient can still extend the elbow. Paralysis of the lateral cord of the brachial plexus would result in loss of the musculocutaneous nerve and the pectoral nerves, which do not mediate extension of the forearm or hand. The medial cord of the brachial plexus branches into the median nerve and ulnar nerve. Neither of these nerves innervates muscles that control extension. The median nerve innervates exors of the forearm and the thenar muscles. The lateral and median pectoral nerves do not extend into the arm and innervate the pectoralis major and minor muscles. GAS 761-763; N 415; McM 139)

38 A 23-year-old male medical student fell asleep in his chair with Netter's Atlas wedged into his axilla. When he awoke in the morning, he was unable to extend his wrist or fingers. Movements of the ipsilateral shoulder joint appear to be normal. Which of the fol- lowing nerves was most likely compressed, producing the symptoms described? A. Lateral cord of the brachial plexus B. Medial cord of the brachial plexus C. Radial nerve D. Median nerve E. Lateral and medial pectoral nerves

A(. The basilic vein can be used for dialysis, especially when the cephalic vein is judged to be too small, as in this case. The basilic vein can be elevated from its position as it passes through the fascia on the medial side of the arm (brachium). The cephalic vein passes more laterally up the limb. The lateral cubital vein is a tributary to the cephalic vein, and the medial cubital vein joins the basilic vein, both of which are rather super cial in position. The medial antebrachial vein courses up the midline of the forearm (antebra- chium) ventrally. GAS 700, 759, 769-770; N 401; McM 148)

39 The kidneys of a 32-year-old woman were failing, and she needed to be placed on dialysis. However, the search in her upper limb for a suitable vein was unex- pectedly dif cult. The major vein on the lateral side of the arm was too small; others were too delicate. Finally, a vein was found on the medial side of the arm that passed through the super cial and deep fascia to join veins beside the brachial artery. Which of the following veins was this? A. Basilic B. Lateral cubital C. Cephalic D. Medial cubital E. Medial antebrachial

A(. When a lumbar puncture is performed, the needle must penetrate the ligamentum flavum, the dura mater, and finally the arachnoid mater to reach the subarachnoid space where the CSF is located. The lumbar cistern is a continuation of the subarachnoid space below the conus medullaris. The pia mater is adherent to the spinal cord, and the posterior longitudinal ligament is attached to the posterior aspect of the vertebral bodies. GAS 106, 114-116; N 161; McM 97)

39 A 24-year-old woman presents with severe headache, photophobia, and stiffness of her back. Physical examination reveals positive signs for meningitis. The attending physician decides to perform a lumbar puncture to determine if a pathogen is in the cerebrospinal fluid (CSF). What is the last structure the needle will penetrate before reaching the lumbar cistern? A. Arachnoid mater B. Dura mater C. Pia mater D. Ligamentum flavum E. Posterior longitudinal ligament

C (. The subarachnoid space, containing the CSF, is located between the pia and the arachnoid mater. Neither the epidural space, the subdural space, nor the pretracheal space contains CSF. Although the central canal, contained within the substance of the spinal cord, does contain CSF, extraction of CSF from this space would result in spinal cord injury. CSF circulates within the subarachnoid space and can be aspirated only from that location. The subdural space is only a potential space between the dura and arach- noid mater. The epidural space contains the epidural fat and Batson's venous plexus and is the site to inject an anesthetic for epidural anesthesia. CSF is not located in the pretracheal space. GAS 106; N 166; McM 97)

40 A 19-year-old presents at the emergency department with high fever, severe headache, nausea, and stiff neck that have persisted for 3 days. The attending physician suspects meningitis and obtains a sample of CSF using a lumbar puncture. From which of the following spaces was the CSF collected? A. Epidural space B. Subdural space C. Subarachnoid space D. Pretracheal space E. Central canal of the spinal cord

A(. General somatic afferent bers are conveyed from the skin of the back via the dorsal primary rami. Communicating rami contain general visceral efferent (sympathetic) bers and general visceral afferent bers of the autonomic nervous system. Ventral primary rami convey mixed spinal nerves to/from all other parts of the body excluding the back, and parts of the head innervated by cranial nerves. The ventral roots contain only efferent (motor) bers. Intercostal nerves are the ventral rami of T1 to T11. The ventral ramus of T12 is the subcostal nerve. GAS 32-48; N 177; McM 98)

41 A 38-year-old man is admitted to the emergency department after a car collision. During physical examination several lacerations to the back are discov- ered. Pain from lacerations or irritations of the skin of the back is conveyed to the central nervous system by which of the following? A. Dorsal primary rami B. Communicating rami C. Ventral primary rami D. Ventral roots E. Intercostal nerves

D(. The conus medullaris is usually located at the L1 to L2 vertebral level; therefore, any choice that contains that region is the correct answer. L3 to L4 is a common location to perform lumbar puncture, but it is caudal to the apex of the conus medullaris. L3 and L4 are caudal to the conus medullaris. T11 is superior to the conus medullaris. GAS 99-110; N 161, 163, 164; McM 97)

42 A 66-year-old woman had been diagnosed with a tumor on her spine. She has started to retain urine and is experiencing rectal incontinence. Both of these symp- toms are signs of conus medullaris syndrome. At which of the following vertebral levels is the tumor probably located? A. L3/L4 B. L3 C. L4 D. T12 to L2 E. T11

A (The anterior axillary (or anterior pectoral) nodes are the first lymph nodes to receive most of the lymph from the breast parenchyma, areola, and nipple. From there, lymph flows through central axillary, apical, and supraclavicular nodes in sequence. The interpectoral Rotter's nodes lie between the pec- toral muscles and are, unfortunately, an alternate route in some patients, speeding the rate of metasta- sis. The parasternal nodes receive lymph from the medial part of the breast and lie along the internal thoracic artery and vein. GAS 748; N 403; McM 179)

42 The 35-year-old woman has a hard breast nodule about 1 cm in diameter slightly above and lateral to the right areola of her right breast. A specifc dye is injected into the tissue around the tumor, and an incision is made to expose the lymphatic vessels draining the area, for the lymphatic vessels to take up the dye, which is visible to the eye. The vessels can then be traced to surgically expose the lymph nodes receiving the lymph from the tumor. Which of the following nodes will most likely first encounter the lymph from the tumor? A. Anterior axillary (pectoral) nodes B. Rotter's interpectoral nodes C. Parasternal nodes along the internal thoracic artery and vein D. Central axillary nodes E. Apical or infraclavicular nodes

D(. The left spinal accessory nerve (CN XI) has been injured distal to the sternocleidomastoid muscle, resulting in paralysis of the trapezius, allowing the shoulder to droop and the superior angle to push out posteriorly. The sternocleidomastoid muscles are intact, as demonstrated by symmetry in strength in turning the head to the right and left. There is no indication of paralysis of the lateral rotators of the shoulder or elbow exors (suprascapular nerve or upper trunk). Thoracodorsal nerve injury would result in paralysis of the latissimus dorsi muscle, an exten- sor, and medial rotator of the humerus. GAS 714; N 413; McM 132)

43 During a ght in a tavern, a 45-year-old male construction worker received a shallow stab wound from a broken beer bottle at a point near the middle of the left posterior triangle of his neck. Upon physical examination, it is observed that the left shoulder is drooping lower than the right shoulder, and the supe- rior angle of the scapula juts out slightly. Strength in turning the head to the right or left appears to be symmetric. Which of the following nerves is most likely injured? A. Suprascapular nerve in the supraspinous fossa B. The terminal segment of the dorsal scapular nerve C. The upper trunk of the brachial plexus D. The spinal accessory nerve in the posterior cervical triangle E. The thoracodorsal nerve in the axilla

D (. Because the meninges and spinal cord are included in the protrusion, the patient's condition is a classic presentation of spina bifida with myelomeningocele. If the protrusion contains only meninges but no CNS tissue, it is known as spina bifida with meningocele. Meningitis is an inflammation of the meninges caused by bacteria, viral, or numerous other irritants (e.g., blood). It does not cause deformation of the vertebrae or result in protrusion of spinal cord contents. Spina bifida occulta is a normally asymptomatic condition in which the vertebral laminae fail to fuse completely during embryologic development. A tuft of hair is commonly seen growing over the affected region (usually lumbar in position). GAS 74; N 160; McM 97)

43 Examination of a 3-day-old male infant reveals protrusion of his spinal cord and meninges from a defect in the lower back. Which of the following describes this congenital anomaly? A. Avulsion of meninges B. Meningitis C. Spina bifida occulta D. Spina bifida with myelomeningocele E. Spina bifida with meningocele

C(. Somatic afferent fibers convey localized pain, typically from the body wall and limbs. Visceral afferents convey autonomic nervous system sensory information. Pain from these fibers will present as dull and diffuse. Somatic efferent fibers convey motor information to skeletal muscle. Sympathetic preganglionic fibers are visceral efferent fibers and do not contain sensory information. Parasympathetic preganglionic fibers are also visceral efferents and do not contain sensory information. GAS 32-48; N 174; McM 97)

44 A 32-year-old mother complains of serious pain in the coccygeal area some days after giving birth. To determine whether the coccyx is involved, a local anesthetic is first injected in the region of the coccyx and then dynamic MRI studies are performed. Physical examination reveals pain with palpation to the region of the coccyx. The local anesthetic is used to interrupt which of the following nerve pathways? A. Visceral afferents B. Somatic efferent C. Somatic afferent D. Sympathetic preganglionic E. Parasympathetic preganglionic

B (. Lateral flexion is the best answer because other movements of the lumbar portion of the vertebral column are very limited due to the orientation of the articular facets. GAS 64-73; N 155; McM 97)

45 During a routine physical examination, a 65-year-old man is tested for ease and flexibility of the movements of his lumbar region. Which of the following movements is most characteristic of the intervertebral joints in the lumbar region? A. Circumduction B. Lateral flexion C. Abduction D. Adduction E. Inversion

D(. Batson's venous plexus, in general, is a valve- less network of veins located in the epidural space of the vertebral canal. The lack of valves can provide a route for the metastasis of cancer (e.g., from prostate or breast to brain) because the ow of blood is bidi- rectional due to local pressures. The length of Bat- son's plexus is irrelevant to the question. B is incorrect because Batson's plexus, in general, does not have valves or one-way movement of blood. Batson's plexus is located within the epidural space, not the subarachnoid or subdural spaces. GAS 102; N 169; McM 88)

46 A 72-year-old man with cancer of the prostate gland presents with loss of consciousness and seizures. A CT scan is performed and a brain tumor is diagnosed. The tumor spread to the brain from the pelvis via the internal vertebral venous plexus (of Batson). What feature of the plexus allows this to happen? A. The internal venous plexus contains the longest veins in the body. B. The internal venous plexus has valves that ensure one-way movement of blood. C. The internal venous plexus is located in the subarachnoid space. D. The internal venous plexus is, in general, valveless. E. The internal venous plexus is located in the subdural space.

D(. The subclavian vein traverses between the clavicle and rst rib and is the most super cial struc- ture to be damaged following a fracture of the clavicle. The subclavian artery runs posterior to the subclavian vein, and though it is in the appropriate location, it would likely not be damaged because of its deep anatomic position. The cephalic vein is a tributary to the axillary vein after ascending on the lateral side of the arm. Its location within the body is too super cial and lateral to the site of injury. The lateral thoracic artery is a branch from the axillary artery that runs lateral to the pectoralis minor. It courses inferior and medial from its point of origin from the axillary artery, and it does not maintain a position near the clavicle during its descent. The internal thoracic artery arises from the rst part of the subclavian artery before descending deep to the costal cartilages. Its point of origin from the subclavian artery is lateral to clavicu- lar injury. Furthermore, its course behind the costal cartilages is quite medial to the clavicular fracture. GAS 694, 736-737; N 415; McM 129)

47 A 29-year-old woman is examined in the emergency department after falling from her balcony. Radio- graphic examination reveals that she has suffered a broken clavicle, with associated internal bleeding. Which of the following vessels is most likely to be injured in clavicular fractures? A. Subclavian artery B. Cephalic vein C. Lateral thoracic artery D. Subclavian vein E. Internal thoracic artery

C (. Spina bifida cystica refers to spina bifida with a meningocele or myelomeningocele and is the correct answer. Cranium bifida could present with meningocele in the skull, but it would not be located in the lower back. Spina bifida occulta is a defect in the formation of the vertebral arches and does not usually present with meningocele. Hemothorax refers to blood accumulation in the pleural space surrounding the lungs. Caudal regression syndrome presents with loss or deformation of the distal part of the spine and/or spinal cord and is not related to a meningocele or myelomeningocele, in general. GAS 74; N 160; McM 97)

49 A maternal serum sample with high alpha-fetoprotein alerted the obstetrician to a possible neural tube defect. Ultrasound diagnosis revealed a myelomeningocele protruding from the back of the child. Which of the following is the most likely diagnosis of this congenital anomaly? A. Cranium bifida B. Spina bifida occulta C. Spina bifida cystica D. Hemothorax E. Caudal regression syndrome

B(. Herpes zoster is a viral disease that remains latent in the dorsal root ganglia of the sensory nerves and when the virus becomes active presents as a painful skin lesion. It is associated only with sensory nerve bers and has no motor involvement. The only answer choice that is solely responsible for sensory innervation is the dorsal root ganglion. GAS 109; N 161-162; McM 95)

5 A 64-year-old man arrived at the clinic with a painful rash and skin eruptions that are localized entirely on one side of his body, closely following the dermatome level of the spinal nerve C7. The patient was diagnosed with the a herpes zoster virus infection known as "shingles." In what structure has the virus most likely proliferated to cause the patient's current condition? A. The sympathetic chain B. The dorsal root ganglion of the C7 spinal nerve C. The lateral horn of the C7 spinal cord segment D. The posterior cutaneous branch of the dorsal primary ramus of C7 E. The ventral horn of the C7 spinal cord segment

C(. The injury has occurred just beyond the third part of the axillary artery. The only collateral arterial channel between the third part of the axillary artery and the brachial artery is between the posterior cir- cum ex humeral and the ascending branch of the profunda brachii, and this anastomotic path is often inadequate to supply the arterial needs of the limb. The posterior circum ex humeral arises from the third part of the axillary artery. It typically anastomoses with a variably small, ascending branch of the pro- funda brachii branch of the brachial artery. The supra- scapular artery anastomoses with the circum ex scapular deep to the infraspinatus. The dorsal scapu- lar artery (passing beneath the medial border of the scapula) has no anastomosis with thoracodorsal within the scope of the injury. The lateral thoracic artery has no anastomoses with the brachial artery. The supreme thoracic artery (from rst part of axil- lary) has no helpful anastomoses with the thoracoac- romial (second part of axillary) (GAS Figs. 7-39 and 7-50). GAS 719-721; N 420; McM 139)

51 The shoulder of a 44-year-old deer hunter had been penetrated by a bolt released from a crossbow. The bolt had transected the axillary artery just beyond the origin of the subscapular artery. A compress is placed on the wound with deep pressure. After a clamp is placed on the bleeding artery, thought is given to the anatomy of the vessel. What collateral arterial path- ways are available to bypass the site of injury? A. Suprascapular with circum ex scapular artery B. Dorsal scapular with thoracodorsal artery C. Posterior circum ex humeral artery with deep brachial artery D. Lateral thoracic with brachial artery E. Supreme thoracic artery with thoracoacromial artery

D(. This question tests anatomic knowledge relating to typical vertebra and the spinal cord. Inter- vertebral disc herniations occur when the nucleus pulposus of the intervertebral disc protrudes through the anulus brosus into the intervertebral foramen or vertebral canal. The most common protrusion is pos- terolaterally, where the anulus brosus is not rein- forced by the posterior longitudinal ligament. The inferior and superior vertebral notches frame the intervertebral foramen, so this is the most likely loca- tion of compression. The denticulate ligaments are lateral extensions of pia mater that anchor to the dura mater, and help maintain the spinal cord in position within the subarachnoid space. The vertebral foramen is the canal through which the spinal cord passes; while this may also be a place of compression, it is not the most likely site of herniation. Articular facets are the locations where vertebral bodies articulate with each other. Intercostovertebral joints are loca- tions where vertebral bodies articulate with ribs. GAS 99-110; N 166; McM 98)

51 A 54-year old woman is admitted to the emergency department due to increasing back pain over the preceding year. MRI reveals that her intervertebral discs have been compressed. It is common for the discs to decrease in size in people older than 40, and this can result in spinal stenosis and disc herniation. At which locations are the spinal nerves most likely to be compressed? A. Between the denticulate ligaments B. As they pass through the vertebral foramen C. Between the superior and inferior articular facets D. Between inferior and superior vertebral notches E. Between the superior and inferior intercostovertebral joints

C (. Caudal anesthesia is used to block the spinal nerves that carry sensation from the perineum. This procedure is commonly used by anesthesiologists to relieve pain during labor and childbirth. Administration of local anesthetic to the epidural space is via the sacral hiatus, which opens between the sacral cornua. The anterior sacral foramina are located on the pelvic surface of the sacrum and are not palpable from a dorsal approach. The posterior sacral foramina and intervertebral foramina are the openings through which sacral nerves exit and are not palpable land- marks. The median sacral crest is cranial to the injec- tion site. GAS 106-110; N 152; McM 90)

52 A 37-year-old pregnant woman is given a caudal epidural block to alleviate pain during vaginal delivery. Caudal epidural blocks involve injection of local anes- thetic into the sacral canal. Which of the following landmarks is most commonly used for the caudal epi- dural block? A. Anterior sacral foramina B. Posterior sacral foramina C. Cornua of the sacral hiatus D. Intervertebral foramina E. Median sacral crest

D(. The sacral cornua lie on either side of the sacral hiatus, from which one can gain access to the sacral canal. This is the best landmark for administra- tion of anesthesia. The ischial tuberosities are more commonly used as landmarks for a pudendal nerve block. The ischial spines are only palpated intravagi- nally. The posterior superior iliac spines, though pal- pable, are not proximal enough for an epidural block within the sacral canal. The coccyx is not part of the sacral canal. GAS 106-110; N 152; McM 90)

53 A 34-year-old pregnant woman in the maternity ward was experiencing considerable pain during labor. Her obstetrician decided to perform a caudal epidural block. What are the most important bony landmarks used for the administration of such anesthesia? A. Ischial tuberosities B. Ischial spines C. Posterior superior iliac spines D. Sacral cornua E. Coccyx

E(. Klippel-Feil syndrome is a congenital defect in which there is a reduction, or extensive fusion of one or more cervical vertebrae. It often manifests as a short, stiff neck with limited motion. Hyperlordosis is an abnormal increase in lumbar curvature. Hyperky- phosis ("hunchback") is an abnormal increase in tho- racic curvature. Scoliosis is a lateral curvature of the spine. Spina bi da can present with deformities in the lumbar region. GAS 76; N 153; McM 83)

55 A 5-year-old boy is admitted to the hospital because of pain in the upper back. Radiologic examina- tion reveals abnormal fusion of the C5 and C6 vertebrae and a high-riding scapula. Which of the following is the most likely diagnosis? A. Lordosis B. Kyphosis C. Scoliosis D. Spina bi da E. Klippel-Feil syndrome

E(. Klippel-Feil syndrome is a congenital defect in which there is a reduction, or extensive fusion of one or more cervical vertebrae. It often manifests as a short, stiff neck with limited motion. Hyperlordosis is an abnormal increase in lumbar curvature. Hyperky- phosis ("hunchback") is an abnormal increase in tho- racic curvature. Scoliosis is a lateral curvature of the spine. Spina bi da can present with deformities in the lumbar region. GAS 76; N 153; McM 83)

55 A 5-year-old boy is admitted to the hospital because of pain in the upper back. Radiologic examination reveals abnormal fusion of the C5 and C6 vertebrae and a high-riding scapula. Which of the following is the most likely diagnosis? A. Lordosis B. Kyphosis C. Scoliosis D. Spina bifida E. Klippel-Feil syndrome

C(. The seventh cervical nerve makes a major contribution to the radial nerve, and this nerve is the prime mover in wrist extension. The dermatome of C7 is in the region described. GAS 745, 787, 790; N 416; McM 153)

57 A 45-year-old woman is admitted to the hospital with neck pain. An MRI examination reveals a herni- ated disc in the cervical region. Physical examination reveals weakness in wrist extension and paraesthesia on the back of her arm and forearm. Which of the fol- lowing spinal nerves is most likely injured? A. C5 B. C6 C. C7 D. C8 E. T1

E(. The axillary sheath is a fascial continuation of the prevertebral layer of the deep cervical fascia extending into the axilla. It encloses the nerves of the neurovascular bundle of the upper limb. Super cial fascia is loose connective tissue between the dermis and the deep investing fascia and contains fat, cutane- ous vessels, nerves, lymphatics, and glands. The buccopharyngeal fascia covers the buccinator muscles and the pharynx mingles with the pretracheal fascia. The clavipectoral muscle invests the clavicle and pec- toralis minor muscle. The axillary fascia is continuous with the pectoral and latissimus dorsi fascia and forms the hollow of the armpit. GAS 721, 731; N 412; McM 361)

60 A 55-year-old male metallurgist had been diag- nosed with carpal tunnel syndrome. To begin the opera- tion, an anesthetic injection into his axillary sheath was given instead of general anesthesia. From which of the following structures does the axillary sheath take origin? A. Super cial fascia of the neck B. Super cial cervical investing fascia C. Buccopharyngeal fascia D. Clavipectoral fascia E. Prevertebral fascia

C(. The spinal accessory nerve (CN XI) arises from the ventral rootlets of C1 to C4 that ascend through the foramen magnum to then exit the cranial cavity through the jugular foramen. It innervates the sternocleidomastoid and trapezius muscles, which function in head rotation and raising of the shoulders. The suprascapular nerve receives bers from C5-6 (occasionally from C4 if the plexus is "pre xed") and innervates the supraspinatus muscle, which is respon- sible for the rst 15 degrees of arm abduction. Erb's point of the brachial plexus is at the union of C5-6 spinal nerves. The long thoracic nerve arises from plexus routes C5, 6, and 7, and supplies the serratus anterior. GAS 714; N 33; McM 132)

61 A 45-year-old woman is admitted to the hospital with neck pain. A computed tomography (CT) scan reveals a tumor on the left side of her oral cavity. The tumor and related tissues are removed and a radical neck surgical procedure is performed. Two months postoperatively the patient's left shoulder droops quite noticeably. Physical examination reveals distinct weak- ness in turning her head to the right and impairment of abduction of her left upper limb to the level of the shoulder. Which of the following structures was most likely injured during the radical neck surgery? A. Suprascapular nerve B. Long thoracic nerve C. Spinal accessory nerve D. The junction of spinal nerves C5 and C6 of the brachial plexus E. Radial nerve

C(. The denticulate ligaments are lateral exten- sions of pia mater that attach to the dura mater between the dorsal and ventral roots of the spinal nerves. These ligaments function to keep the spinal cord in the midline position. The posterior longitudi- nal ligament supports the posterior aspect of the ver- tebrae within the vertebral canal. The tentorium cerebelli is a layer of dura mater that supports the occipital lobes of the cerebral hemispheres and covers the cerebellum. The ligamentum avum helps main- tain upright posture by connecting the laminae of two adjacent vertebrae. The nuchal ligament is a thicken- ing of the supraspinous ligaments extending from the C7 vertebra to the external occipital protuberance. GAS 99-110; N 165; McM 94-96)

61 A 23-year-old college student is admitted to the emergency department after jumping from a 50-foot high waterfall. The MRI of his back reveals a lateral shift of the spinal cord to the left. Which of the following structures has most likely been torn to cause the deviation? A. Posterior longitudinal ligament B. Tentorium cerebelli C. Denticulate ligaments D. Ligamentum flavum E. Nuchal ligament

A(. The quadrangular space is bordered medially by the long head of the triceps brachii muscle, later- ally by the surgical neck of the humerus, superiorly by the teres minor and subscapularis muscles, and inferiorly by the teres major muscle. Both the axillary nerve and posterior circum ex humeral vessels tra- verse this space. The other structures listed are not contained within the quadrangular space. The cephalic vein is located in the deltopectoral triangle, and the radial nerve is located in the triangular interval. GAS 718-720, 730; N 413; McM 139)

63 A 35-year-old male body builder has enlarged his shoulder muscles to such a degree that the size of the quadrangular space is greatly reduced. Which of the following structures would most likely be compressed in this condition? A. Axillary nerve B. Anterior circum ex humeral artery C. Cephalic vein D. Radial nerve E. Subscapular artery

C(. The weakness in shoulder movement results from denervation of the teres minor and deltoid by the axillary nerve, which passes through the quadran- gular space. Quadrangular space syndrome occurs when there is hypertrophy of the muscles that border the quadrangular space or brosis of portions of the muscles that are in contact with the nerve. GAS 718; N 413; McM 102)

63 A 45-year old woman is admitted to the outpatient clinic for shoulder pain. During physical examination she presents with weakened shoulder movements. Radiologic examination reveals signs of quadrangular space syndrome, causing weakened shoulder move- ments. Which of the following nerves is most likely affected? A. Suprascapular B. Subscapular C. Axillary D. Radial E. Ulnar

B(. The (great radicular) artery of Adamkiewicz is important for blood supply to anterior and posterior spinal arteries. The location of this artery should be noted during surgery because damage to it can result in dire consequences, including paraplegia (loss of all sensation and voluntary movement inferior and at the level of the injury). Injury to the left vertebral artery would not be likely due its superior location to the surgical site. Ligation of the posterior spinal artery would not occur because of its protected location inside the vertebral canal. Transection of the conus medullaris of the spinal cord would not occur as this structure is located at L1, L2 levels and is, again, protected inside the vertebral canal. Division of the thoracic sympathetic chain would not be likely as the symptoms described include limb paralysis, which would not be a consequence of sympathetic disrup- tion (GAS Fig. 2-49A). GAS 100-101; N 167)

65 A 58-year-old man in the intensive care ward exhibited little voluntary control of urinary or fecal activity following repair of an abdominal aortic aneurysm. In addition, physical examination revealed widespread paralysis of his lower limbs. These functions were essentially normal prior to admission to the hospital. The most likely cause of this patient's problems is which of the following? A. Injury to the left vertebral artery B. Injury of the great radicular artery (of Adamkiewicz) C. Ligation of the posterior spinal artery D. Transection of the conal segment of the spinal cord E. Division of the thoracic sympathetic chain

C(. Scoliosis can be a secondary condition in such disorders as muscular dystrophy and polio in which abnormal muscle does not keep the normal alignment of the vertebral column and results in a lateral curvature. Hyperlordosis is increased second- ary curvature of the lumbar region. It can be caused by stress on the lower back and is quite common during late pregnancy. Hyperkyphosis is increased primary curvature of the thoracic regions and pro- duces a hunchback deformity. It can be secondary to tuberculosis, producing a "gibbus deformity," which results in angulated kyphosis at the lesion site. Spina bi da is a congenital defect and would not present as a result of muscular dystrophy or polio. Osteoarthritis most commonly presents with age from normal "wear and tear." It is highly unlikely in a 23-year-old woman. GAS 75; N 153; McM 83)

66 A 23-year-old woman is admitted to the hospital due to back pain. Radiologic examination reveals that she suffers from a clinical condition affecting her vertebral column. Her history reveals that she suffered from polio and has a muscular dystrophy. Which of the following conditions of the vertebral column will most likely be present in this patient? A. Hyperlordosis B. Hyperkyphosis C. Scoliosis D. Spina bifida E. Osteoarthritis

A(. The circum ex scapular artery passes through the triangular space after arising from the subscapu- lar artery. It provides super cial branches to the overlying latissimus dorsi, whereas its deep portion passes into the infraspinous fossa to anastomose with the suprascapular artery. The dorsal scapular artery passes between the ventral rami of the brachial plexus and then deep to the medial border of the scapula. The transverse cervical artery arises from the thyro- cervical trunk at the root of the neck and can provide origin for a dorsal scapular branch. The lateral tho- racic and thoracoacromial arteries are branches of the second part of the axillary artery and provide no supply to the latissimus dorsi. GAS 721; N 414; McM 141)

67 A 41-year-old woman is scheduled for a latissimus dorsi muscle ap to cosmetically augment the site of her absent left breast after mastectomy. Part of the latissimus dorsi muscle is advanced to the anterior thoracic wall, based upon arterial supply provided in part by the artery that passes through the triangular space of the axilla. Which artery forms the vascular base of this flap? A. Circum flex scapular artery B. Dorsal scapular artery C. Transverse cervical artery D. Lateral thoracic artery E. Thoracoacromial artery

D(. A lumbar puncture is performed by taking a sample of CSF from the lumbar cistern (the subarachnoid space below the spinal cord) between vertebrae L4 and L5 or sometimes between L3 and L4. It is done in this region because the spinal cord typically ends at the level of L1 to L2 and the dural sac ends at the level of S2. Therefore, it is the safest place to do the procedure because it lies between these areas and the risk of injuring the spinal cord is minimized. (Remember in children the cord ends more caudally.) GAS 116; N 160-161; McM 97)

7 A 3-year-old boy is brought by his mother to the emergency department with severe headache, high fever, malaise, and confusion. Radiologic and physical examinations reveal that the patient suffers from meningitis. A lumbar puncture is ordered to confirm the diagnosis. Which vertebral level is the most appropriate location for the lumbar puncture? A. T12 to L1 B. L1 to L2 C. L2 to L3 D. L4 to L5 E. L5 to S1

E (. Scoliosis (severe lateral curvature of the spine) in the patient is causing compression or stretching of the T1 spinal nerve ramus by the first rib as the nerve ascends to join C8 and form the lower trunk of the brachial plexus. T1 provides sensation for the medial side of the forearm, via the medial antebrachial cuta- neous nerve from the medial cord of the brachial plexus. T1 is the principal source of motor supply to all of the intrinsic muscles in the palm. Its dysfunction affects all fine motor movements of the digits. Long flexors of the fingers are intact; therefore, the median nerve and ulnar nerve are not injured. The extensors of the wrist are functional; therefore, the radial nerve is not paralyzed. The only sensory disturbance is that of the T1 dermatome. GAS 695-700, 744-745; N 161; McM 94)

72 Fine motor function in the right hand of a 14-year- old girl with scoliosis since birth appeared to be quite reduced, including opposition of the thumb, abduction and adduction of the digits, and interphalangeal joint extension. Radiography confirmed that her severe scoliosis was causing marked elevation of the right first rib. Long flexor muscles of the hand and long extensors of the wrist appear to be functioning within normal limits. There is notable anesthesia of the skin on the medial side of the forearm; otherwise, sensory function in the limb is intact. Which of the following neural structures is most likely impaired? A. Median nerve B. Middle trunk of the brachial plexus C. Radial nerve D. Lower trunk of the brachial plexus E. T1 ventral ramus

C(. Somatic afferent bers convey localized pain, typically from the body wall and limbs and the cell bodies are found in the dorsal root ganglia. The dorsal horn is found at all spinal cord levels and is comprised of sensory nuclei that receive and process incoming somatosensory information. The lateral horn com- prises autonomic neurons innervating visceral and pelvic organs. The sympathetic chain ganglia deliver the sympathetic information to the body. White rami communicans carry preganglionic sympathetic bers and are called white because the bers it contains are myelinated. GAS 31-48; N 177; McM 96)

74 A 65-year-old woman who has been otherwise well presents to her physician with complaints of a group of painful blisters over her back in the distribu- tion of the T9 dermatome. She noticed that a few days prior to the eruption of the blisters she experienced an intense burning sensation over her skin. She was diag- nosed with herpes zoster (shingles). Where are the neural cell bodies located that are responsible for the pain sensation? A. Dorsal horn B. Lateral horn C. Dorsal root ganglia D. Sympathetic chain ganglia E. White rami communicans

E(. Scoliosis is de ned as a lateral deviation of the spinal column to either side and is often associ- ated with a "rib-hump" as seen on examination when bending forward to touch the toes. Hyperkyphosis is an increased primary curvature of the spinal column. This curvature is associated with thoracic and sacral regions and is most likely this patient's clinical condi- tion. Hyperlordosis is the increased secondary curva- ture affecting the cervical and lumbar regions. GAS 75; N 153; McM 83)

76 A 16-year-old girl is sent for a presports physical examination prior to the beginning of her school year. She has no medical complaints or any clinical past history. On physical examination, the physician notices one shoulder is higher than the other. The student is then asked to bend forward at the waist to touch her toes. This maneuver results in a posterior bulging of the ribs on the right side. Which one of the following is most likely diagnosis? A. Kyphosis B. Spondylosis C. Lordosis D. Spondylolisthesis E. Scoliosis

D(. Cells from a speci c somite develop into the dermis of the skin in a precise location, somatic sensory bers originally associated with that somite enter the posterior region of the spinal nerve. The somatic sensory (afferent) bers convey localized pain, typically from the body wall and limbs and the cell bodies are found in the dorsal root ganglia. The lateral horn comprises autonomic neurons innervating visceral and pelvic organs. The lateral horn comprises autonomic neurons innervating visceral and pelvic organs. The anterior horn contains motor neurons that supply muscles of the body wall and the limbs. GAS 32-48; N 162; McM 96)

77 A 60-year-old man has been feeling sharp pains over his left lower chest and back for several days. A rash of red erupted vesicles is seen at the left border of the sternum just at the level of the xiphoid process. Antiviral treatment is given for herpes zoster. Which of the following locations will contain the neural cell bodies responsible for the painful sensation? A. Ventral horn of T6 spinal cord segment B. Lateral horn of T6 spinal cord segment C. Dorsal root ganglion of T4 spinal nerve D. Dorsal root ganglion of T6 spinal nerve E. Dorsal root ganglion of T10 spinal nerve

E(. In the cervical region, spinal nerves exit the vertebral column above their named vertebrae. From the thoracic region and below the spinal nerves exit the vertebral column below their named vertebrae. GAS 107-110; N 161; McM 94)

78 A 70-year-old man with prostate cancer is experi- encing sharp shooting pains radiating from his neck into the upper limb. An MRI of his spine demonstrates a small metastatic mass in the cervical region extending into the left intervertebral foramen between C6 and C7. The intervertebral discs appear normal. Which neural structure is most likely being compressed by the meta- static mass to account for the pain? A. C8 spinal nerve B. Dorsal horn of C6 spinal cord segment C. C6 spinal nerve D. Dorsal horn of C7 spinal nerve E. C7 spinal nerve

C(. Lumbar puncture is generally performed at the level of L4, L5. The spinal cord ends at the level of L1/L2 in adults and at the level of L2/L3 in newborns. GAS 103-104, 106; N 161; McM 97)

79 A 3-day-old girl develops a fever. She is irritable and not feeding. As part of the workup for fever of unknown origin, a lumbar puncture is performed. This puncture must be done below the spinal cord which usually ends at which vertebral level in a patient of this age? A. L1 B. S1 C. L3 D. S3 E. L5

C(. Lumbar puncture is generally performed at the level of L4, L5. The spinal cord ends at the level of L1/L2 in adults and at the level of L2/L3 in newborns. GAS 103-104, 106; N 161; McM 97)

79 A 3-day-old girl develops a fever. She is irritable and not feeding. As part of the workup for fever of unknown origin, a lumbar puncture is performed. This puncture must be done below the spinal cord which usually ends at which vertebral level in a patient of this age? A. L1 B. S1 C. L3 D. S3 E. L5

B(. The highest points of the iliac crests are used as a landmark for locating the position of L4 to L5 for a lumbar puncture; they are identified and traced medially toward the vertebral column (Tuffier's line). The inferior angles of the scapulae lie at vertebral level T7; the lowest ribs lead one to T12; the sacral hiatus is located lower at the distal portion of the sacrum; the posterior inferior iliac spines lie below S2. GAS 106, 114-116; N 160-161; McM 97)

8 When a lumbar puncture is performed to sample cerebrospinal fluid, which of the following external landmarks is the most reliable to determine the position of the L4 vertebral spine? A. The inferior angles of the scapulae B. The highest points of the iliac crests C. The lowest pair of ribs bilaterally D. The sacral hiatus E. The posterior inferior iliac spines

A(. The anterior longitudinal ligament is a strong brous band that covers and connects the anterolat- eral aspect of the vertebrae and intervertebral discs; it maintains stability and prevents hyperextension. It can be torn by cervical hyperextension. The liga- mentum avum helps maintain upright posture by connecting the laminae of two adjacent vertebrae. The posterior longitudinal ligament runs within the vertebral canal supporting the posterior aspect of the vertebrae and prevents hyper exion. The anulus brosus is the outer brous part of an intervertebral disc. The interspinous ligament connects adjacent spinous processes. GAS 80-83; N 159; McM 94)

81 While waiting in his car at a stop sign, a 28-year-old man was rear-ended by a van, resulting in neck hyperextension. He was admitted to the emergency department and a whiplash injury was diagnosed. The next day his neck was stiff and painful. Which structure was most likely damaged to cause the pain? A. Anterior longitudinal ligament B. Posterior longitudinal ligament C. Ligamentum flavum D. Intervertebral disc E. Supraspinous ligament

D(. In a lesion of the lower trunk of the brachial plexus, or the C8 and T1 ventral rami, there is sensory loss on the medial forearm and the medial side of hand (dorsal and ventral). The medial cord is an extension of the lower trunk. The medial cord gives origin to the medial antebrachial cutaneous nerve, which supplies the T1 dermatome of the medial side of the antebrachium. The lower lateral brachial cuta- neous nerve arises from the radial nerve, C5 and C6. The musculocutaneous nerve arises from the lateral cord, ending in the lateral antebrachial cutaneous nerve, with C5 and C6 dermatome bers. The inter- costobrachial nerve is the lateral cutaneous branch of the T2 ventral primary ramus and supplies skin on the medial side of the arm. The median nerve distrib- utes C6 and C7 sensory bers to the lateral part of the palm, thumb, index, long nger, and half of the ring finger. GAS 738-745; N 416; McM 138)

82 A 19-year old man fell from a cliff when he was hiking in the mountains. He broke his fall by grasping a tree branch, but he suffered injury to the C8 to T1 spinal nerve ventral rami. Sensory tests would thereaf- ter con rm the nature of his neurologic injury by the sensory loss in the part of the limb supplied by which of the following? A. Lower lateral brachial cutaneous nerve B. Musculocutaneous nerve C. Intercostobrachial nerve D. Medial antebrachial cutaneous nerve E. Median nerve

D(. The boundaries of an intervertebral foramen (clockwise) include the following: the superior margin (roof) is formed by the inferior vertebral notch of the vertebra above, the anterior margin by the interverte- bral disc between the vertebral bodies of the adjacent vertebrae, the inferior margin ( oor) by the superior vertebral notch of the vertebra below, and the poste- rior margin by the zygapophysial (facet) joint of the adjacent vertebrae. Each pedicle contains superior and inferior vertebral notches. GAS 66-72; N 158; McM 98)

82 A 38-year-old man who is a professional golfer complains of chronic lower back pain with radiating pain to the heel. The pain is so debilitating that he now has trouble ambulating. MRI of the lower back reveals severe narrowing of an intervertebral foramen (IVF), which has caused compression of the exiting nerve root. Surgery is required to correct the problem. During surgery the neurosurgeon carefully accesses the IVF using a lateral approach and shaves bone off the superior margin (roof) of the IVF to decompress the exiting nerve root. Which of the following vertebral bony features is the neurosurgeon most likely shaving off? A. Superior articular process B. Lamina C. Inferior articular process D. Pedicle E. Spinous process

C(. The first branch of the lateral pectoral nerve is typically the only source of motor supply to the clavicular head of the pectoralis major muscle. If it is injured (as in this case of an iatrogenic injury when the infraclavicular nodes were removed), this part of the muscle undergoes atrophy, leaving an infraclavicular cosmetic deficit. The remainder of the lateral pectoral nerve joins the medial pectoral nerve in a neural arch that provides motor supply to the remaining parts of the pectoralis major and the pectoralis minor. Physical examination reveals no obvious motor or sensory de cits. Loss of the medial pectoral nerve would have no effect on the clavicular head of pecto- ralis major and might not be discernible. Injury to the lateral cord would lead to loss not only of all of the lateral pectoral nerve but also the musculocutaneous nerve, resulting in biceps brachii and brachialis paral- ysis and lateral antebrachial sensory loss. GAS 724, 742; N 415; McM 141)

83 The mastectomy procedure on a 52-year-old woman involved excision of the tumor and removal of lymph nodes, including the pectoral, central axillary, and infraclavicular groups. Six months after her mastectomy, the patient complains to her personal physician of an unsightly deep hollow area inferior to the medial half of the clavicle, indicating a significant area of muscle atrophy and loss. She states that the disfigurement has taken place quite gradually since her mastectomy. Physical examination reveals no obvious motor or sensory deficits. What was the most likely cause of the patient's cosmetic problem? A. Part of the pectoralis major muscle was cut and removed in the mastectomy B. The pectoralis minor muscle was removed entirely in the surgery C. A branch of the lateral pectoral nerve was cut D. The medial pectoral nerve was cut E. The lateral cord of the brachial plexus was injured

C(. The most superiorly positioned intervertebral disc is between the C2 to C3 vertebrae. In the cervical region the spinal nerves exit superior to their corre- sponding vertebrae and take a somewhat horizontal path. The C3 nerve therefore exits through the C2 to C3 intervertebral foramen and would be affected by a posterolateral disc herniation at this level. The C1 nerve exists between the C1 vertebra and the occipital bone of the cranium and would not be affected. The C2 nerve passes superior to the second vertebra and would not be affected by a herniated disc between C2 and C3. C4 and C5 both exit superior to their corre- sponding vertebrae which is below the level of the herniated disc and will therefore not be affected. GAS 79, 108; N 161; McM 94)

83 The following statement was found in the radiology report of a magnetic resonance imaging scan of the cervical spine: "An acute posterolateral herniation of the most superiorly located intervertebral disc is located within the corresponding intervertebral foramen and compressing the exiting nerve." Which of the following nerves was the radiologist most likely referring to in the report? A. C1 B. C2 C. C3 D. C4 E. C5

C(. C7 is the main spinal nerve that contributes to the radial nerve and innervates the triceps brachii. Absence of the triceps re ex is usually indicative of a C7 radiculopathy or injury. GAS 745-746, 756; N 416; McM 144)

85 A 43-year-old man is admitted to the hospital, having suffered a whiplash injury when his compact automobile was struck from behind by a sports utility vehicle. MRI examination reveals some herniation of a disc in the cervical region. Physical examination reveals that the patient has lost elbow extension; there is absence of his triceps re ex and loss of extension of the metacarpophalangeal joints on the ipsilateral side. Which of the following spinal nerves is most likely affected? A. C5 B. C6 C. C7 D. C8 E. T1

C(. The anterior spinal artery lies in the anterior median ssure and would likely be compressed by the tumor. This artery is formed superiorly by the union of two branches that directly arise from the vertebral arteries. Ascending cervical artery is not found within the vertebral canal and does not contribute to the blood supply of the spinal cord. The segmental spinal arteries follow the spinal nerves and provide the seg- mental medullary vessels which run along the dorsal and ventral roots to supply the lateral aspect of the spinal cord. There are two posterior spinal arteries, each are located in the posterolateral sulcus on the posterior aspect and have only small branches to the direct area. GAS 100-101; N 167; McM 94)

86 An MRI scan of the thoracic spine of a 68-year-old man with acute midback pain revealed a large tumor arising from the anterior median fissure of the spinal cord at the level of the T3 vertebra. The artery compressed by the tumor is most likely formed superiorly by direct branches from which of the following arteries? A. Ascending cervical B. Segmental medullary C. Vertebral D. Segmental spinal E. Posterior spinal

C(. There are seven (7) cervical vertebrae and eight (8) cervical spinal nerves. Nerves C1 to C7 exit superior to their corresponding vertebrae, whereas nerve C8 exits inferiorly to the C7 vertebra. The nerves of the thoracic and subsequent regions all exit inferior to their corresponding vertebrae. GAS 99-100, 108; N 158; McM 94)

88 In a report of a radiograph of the cervical spine the radiologist wrote the following: "Severe narrowing of the C7-T1 intervertebral foramen (IVF) on the left." Which nerve was most likely compressed as a result of this finding? A. C6 B. C7 C. C8 E. T1

C(. When a child is born only one curvature is present in the vertebral column, the primary curva- ture, which is concave anteriorly and termed kypho- sis. During postnatal development two additional curvatures form, secondary curvatures, which are convex anteriorly and termed lordosis. The rst forms in response to the child lifting its head and is in the cervical spine, the second forms once the child is sitting and completes once the child starts to walk. Thoracic kyphosis is the normal curvature with which we are born and cervical lordosis is associated with the neck and develops much earlier on. Cervical kyphosis and thoracic lordosis would both be consid- ered abnormal curvatures in a child of this age. GAS 54; N 153; McM 83)

89 A mother brought her 15-month-old previously healthy child to the pediatrician and nervously told the physician that her child now had "a curvature in her low back." The mother stated that this curvature was not present previously and that she noticed it when the child started standing and walking. The physician examined the child and then reassured the mother that the spinal curvature was normal. Which curvature did the mother most likely observe? A. Thoracic kyphosis B. Cervical lordosis C. Lumbar lordosis D. Cervical kyphosis E. Thoracic lordosis

D (. Lymphatic drainage of the breast is typically to the axillary nodes, more speci cally to the anterior (pectoral) nodes. Lymphatic vessels from the pectoral nodes continue into the central axillary nodes, the drainage of which passes farther into the apical node, just inferior to the clavicle in the deltopectoral trian- gle. From these nodes lymph passes to the "sentinel," or scalene, nodes and the subclavian lymph trunk. The lateral and posterior axillary nodes do not nor- mally receive lymph drainage from the breast but do receive lymph from the upper limb. (This is the reason for the edema of the upper limb that occurs after a mastectomy, in which there may be a total removal of axillary lymph nodes.) GAS 140-141; N 182; McM 179)

91 A 35-year-old woman is admitted to a surgical ward with a palpable mass in her right breast and swollen lymph nodes in the ipsilateral axilla. Radiologic studies and biopsy reveal carcinoma of the breast. Which group of axillary lymph nodes is the first to receive lymph drainage from the secretory tissue of the breast and therefore most likely to contain metastasized tumor cells? A. Lateral B. Central C. Apical D. Anterior (pectoral) E. Posterior (subscapular)

C(. The long thoracic nerve was injured during the axillary dissection, resulting in paralysis of the serratus anterior muscle. The serratus anterior is important in rotation of the scapula in raising the arm above the level of the shoulder. Its loss results in protrusion of the medial border ("winging" of the scapula), which is more obvious when one pushes against resistance. The long thoracic nerve arises from the ventral rami of C5, C6, and C7. The upper trunk (C5, C6) supplies rotator and abductor muscles of the shoulder and elbow exors. The posterior division of the middle trunk contains C7 bers for distribution to extensor muscles; likewise, the posterior cord supplies extensors of the arm, forearm, and hand. The lateral cord (C5, C6, and C7) gives origin to the lateral pec- toral nerve, the musculocutaneous nerve, and the lateral root of the median nerve. There is no sensory loss in the limb in this patient; injury to any of the other nerve elements listed here would be associated with speci c dermatome losses. GAS 726-727; N 413; McM 138)

93 Several weeks after surgical dissection of her left axilla for the removal of lymph nodes for staging and treatment of her breast cancer, a 32-year-old woman was told by her general physician that she had "winging" of her left scapula when she pushed against resistance during her physical examination. She told the physician that she had also experienced dif culty lately in raising her left arm above her head when she was combing her hair. In a subsequent consult visit with her surgeon, she was told that a nerve was acci- dentally injured during the diagnostic surgical proce- dure and that this produced her scapular abnormality and inability to raise her arm normally. What was the origin of this nerve? A. The upper trunk of her brachial plexus B. The posterior division of the middle trunk C. Ventral rami of the brachial plexus D. The posterior cord of the brachial plexus E. The lateral cord of the brachial plexus

C(. The long thoracic nerve arises from the C5, C6, and C7 ventral rami and innervates the serratus anterior muscle. Injury of this nerve will result in a characteristic winged scapula. A is the lateral pec- toral nerve, which innervates the pectoralis major muscle. B is the suprascapular nerve, which inner- vates the supraspinatus and infraspinatus muscles. D is the thoracodorsal nerve, which innervates latissi- mus dorsi. E is the lower subscapular nerve that innervates the lower part of the subscapularis muscle and the teres major. GAS 728, 738, 804; N 185; McM 138)

94 A patient who has undergone a radical mastec- tomy with extensive axillary dissection exhibits winging of the scapula when she pushes against resistance on an immovable object, such as a wall. Injury of which of the following nerves would result in this condition? A. A B. B C. C D. D E. E

A(. The axillary nerve is a direct branch of the posterior cord and wraps around the surgical neck of the humerus to innervate the teres minor and the deltoid muscles. With this anatomic arrangement, the axillary nerve is tightly "tethered" to the proximal humerus. When the head of the humerus is dislo- cated, it often puts traction on the axillary nerve. GAS 718-719; N 413; McM 142)

95 A 24-year-old female basketball player is admitted to the emergency department after an injury to her shoulder. Radiographic examination reveals a shoulder dislocation. What is the most commonly injured nerve in shoulder dislocations? A. Axillary B. Radial C. Median D. Ulnar E. Musculocutaneous

A (. The radicular arteries are branches of the segmental spinal arteries. They occur at every verte- bral level and follow and provide blood supply to the anterior and posterior roots. A space occupying lesion that compresses the posterior roots will also compress the arteries that supply them. The segmental spinal arteries are feeder arteries that reinforce the blood supply to the spinal cord and arise from the vertebral and deep cervical arteries in the neck, the posterior intercostals in the thorax, and the lumbar arteries in the abdomen. The anterior and posterior spinal arter- ies arise from the vertebral artery and supply the spinal cord directly. The segmental medullary arteries are also branches of the segmental spinal arteries that anastomose directly with the anterior and posterior spinal arteries. GAS 100-101; N 168; McM 96)

97 A 35-year-old man underwent a laminectomy of the T8 to T9 vertebrae. During the surgery, the neurosurgeon observed that the posterior roots were compressed at that level due to a space-occupying lesion. Which of the following arteries was most likely directly compressed by the lesion? A. Radicular B. Segmental spinal C. Segmental medullary D. Anterior spinal E. Posterior spinal

C(. The supraspinatus muscle inserts on the greater tubercle of the humerus and is said to initiate abduction of the arm at the shoulder. It is supplied principally by spinal nerve C5. The subscapularis muscle is the only muscle that inserts on the lesser tubercle. The subscapularis muscle is innervated by the upper and lower subscapular nerves. The teres minor muscle takes origin from the lateral border of the scapula; the teres major muscle takes origin from the region of the inferior angle and the lateral border of the scapula. GAS 717; N 411; McM 115)

97 After an orthopedic surgeon examined the MRI of the shoulder of a 42-year-old woman he informed her that the supraspinatus tendon was injured and needed to be repaired surgically. Which of the following is true of the supraspinatus muscle? A. It inserts onto the lesser tubercle of the humerus. B. It initiates adduction of the shoulder. C. It is innervated chie y by the C5 spinal nerve. D. It is supplied by the upper subscapular nerve. E. It originates from the lateral border of the scapula.

C (. The patient is suffering from thoracic outlet syndrome, involving neural and vascular elements. This results from any condition that decreases the dimensions of the superior thoracic aperture (the formal name of the thoracic outlet). It could be a result of a cervical rib, accessory muscles, and/or atypical connective tissue bands at the root of the neck. In this case, symptoms involve the arm, forearm, and hand. Paraesthesia along the medial forearm and hand and atrophy of long exors and intrinsic muscles point to a possible compression or traction problem of the lower trunk (C8, T1) rather than a lesion of either the median or ulnar nerve. The lateral palm has no sensory problem, which tends to rule out median nerve involvement. Changes in the radial artery pulse point to vascular compression. Erb-Duchenne paraly- sis of the upper trunk would affect proximal limb functions, such as arm rotation, abduction, and so on. This lesion is on the left side, so the brachiocephalic artery could not be involved because it arises from the right side of the aortic arch; moreover, it would not compress the brachial plexus. Carpal tunnel syn- drome would not explain the problems of the forearm and medial hand, or the long exor atrophy. An iso- lated medial cord lesion would not explain the atrophy of all long exors and intrinsic muscles and does not explain the radial pulse characteristics. The ischemic pain in the arm is due to vascular compression. GAS 150; N 183; McM 138)

99 A 22-year-old woman visits the outpatient clinic with pain in her left upper limb. She has a long history of pain in this limb and difficulty with fine motor tasks of the hand. Physical examination reveals paraesthesia along the medial surface of the forearm and palm and weakness and atrophy of gripping muscles ("long exors") and the intrinsic muscles of the hand. The radial pulse is diminished when her neck is rotated to the ipsilateral side (positive Adson's test). What is the most likely diagnosis? A. Erb-Duchenne paralysis B. Aneurysm of the brachiocephalic artery, with plexus compression C. Thoracic outlet syndrome D. Carpal tunnel syndrome E. Injury to the medial cord of the brachial

A(. In the cervical region, the spinal nerve exits in the intervertebral foramen above the correspond- ingly named vertebrae. Therefore, the C3 spinal nerve exits above the C3 vertebrae and lies directly below the C2 vertebrae. GAS 106-109; N 161; McM 94)

99 The following statement is written in the radiol- ogy report of an MRI scan of the cervical spine: "A large osteophyte is emanating from the posterolateral area of the vertebral body of the vertebra immediately above the C3 nerve root and is severely compressing the C3 nerve root." The osteophyte is most likely emanating from which of the following vertebrae? A. C2 B. C3 C. C4 D. C5 E. C1


Conjuntos de estudio relacionados

Accounting: Chp 1 &2 Practice questions

View Set

Management Chapter 8 Learn Smart

View Set

Pharm 125 Pain and Inflammation Test

View Set

Chapter 6 - Elasticity: The Responsiveness of Demand and Supply

View Set

Chapter 11 Exceptions and Advanced File I/O

View Set

CO2 Ventilatory Response Curve APEX (question 19) respiratory material

View Set

Substitution in Algebra, Combine Like Terms, Algebraic Expressions

View Set